You are on page 1of 123

EC-GATE-2014 PAPER-01| www.gateforum.

com

Q. No. 1 – 5 Carry One Mark Each

1. Choose the most appropriate phrase from the options given below to complete the following
sentence.
The aircraft_______ take off as soon as its flight plan was filed.
(A) is allowed to (B) will be allowed to
(C) was allowed to (D) has been allowed to
Answer: (C)

2. Read the statements:


All women are entrepreneurs.
Some women are doctors
Which of the following conclusions can be logically inferred from the above statements?
(A) All women are doctors (B) All doctors are entrepreneurs
(C) All entrepreneurs are women (D) Some entrepreneurs are doctors
Answer: (D)

3. Choose the most appropriate word from the options given below to complete the following
sentence.
Many ancient cultures attributed disease to supernatural causes. However, modern science
has largely helped _________ such notions.
(A) impel (B) dispel (C) propel (D) repel
Answer: (B)

4. The statistics of runs scored in a series by four batsmen are provided in the following table,
Who is the most consistent batsman of these four?

Batsman Average Standard deviation


K 31.2 5.21
L 46.0 6.35
M 54.4 6.22
N 17.9 5.90

(A) K (B) L (C) M (D) N


Answer: (A)
Exp: If the standard deviation is less, there will be less deviation or batsman is more consistent

5. What is the next number in the series?


12 35 81 173 357 ____
Answer: 725

 India’s No.1 institute for GATE Training  1 Lakh+ Students trained till date  65+ Centers across India
1
EC-GATE-2014 PAPER-01| www.gateforum.com

Exp: 12 35 81 173 357 ________

23 46 92 184 368
difference

357
⇒ 368
725
Q. No. 6 – 10 Carry One Mark Each

6. Find the odd one from the following group:


W,E,K,O I,Q,W,A F,N,T,X N,V,B,D
(A) W,E,K,O (B) I,Q,W,A (B) F,N,T,X (D) N,V,B,D
Answer: (D)
Exp: 1 Q W A
W E K O F N T X N V B D

8 6 4 8 6 4 8 6 4 8 6 2

Difference of position: D

7. For submitting tax returns, all resident males with annual income below Rs 10 lakh should fill
up Form P and all resident females with income below Rs 8 lakh should fill up Form All
people with incomes above Rs 10 lakh should fill up Form R, except non residents with
income above Rs 15 lakhs, who should fill up Form S. All others should fill Form T. An
example of a person who should fill Form T is
(A) a resident male with annual income Rs 9 lakh
(B) a resident female with annual income Rs 9 lakh
(C) a non-resident male with annual income Rs 16 lakh
(D) a non-resident female with annual income Rs 16 lakh
Answer: (B)
Exp: Resident female in between 8 to 10 lakhs haven’t been mentioned.

8. A train that is 280 metres long, travelling at a uniform speed, crosses a platform in 60 seconds
and passes a man standing on the platform in 20 seconds. What is the length of the platform
in metres?
Answer: 560
Exp: For a train to cross a person, it takes 20 seconds for its 280m.
So, for second 60 seconds. Total distance travelled should be 840. Including 280 train length
so length of plates =840-280=560

 India’s No.1 institute for GATE Training  1 Lakh+ Students trained till date  65+ Centers across India
2
EC-GATE-2014 PAPER-01| www.gateforum.com

9. The exports and imports (in crores of Rs.) of a country from 2000 to 2007 are given in the
following bar chart. If the trade deficit is defined as excess of imports over exports, in which
year is the trade deficit 1/5th of the exports?

120
Exports Im ports
110
100
90
80
70
60
50
40
30
20
10
0 2006
2000 2001 2002 2003 2004 2005 2007

(A) 2005 (B) 2004 (C) 2007 (D) 2006


Answer: (D)
imports − exp orts 10 1
Exp: 2004, = =
exp orts 70 7
26 2
2 0 0 5, =
76 7
20 1
2 0 0 6, =
100 5
10 1
2007, =
100 11

10. You are given three coins: one has heads on both faces, the second has tails on both faces,
and the third has a head on one face and a tail on the other. You choose a coin at random and
toss it, and it comes up heads. The probability that the other face is tails is
(A) 1/4 (B) 1/3 (C) 1/2 (D) 2/3
Answer: (B)

 India’s No.1 institute for GATE Training  1 Lakh+ Students trained till date  65+ Centers across India
3
EC-GATE-2014 PAPER-01| www.gateforum.com

Q. No. 1 – 25 Carry One Mark Each

1. For matrices of same dimension M, N and scalar c, which one of these properties DOES NOT
ALWAYS hold?
(A) (MT)T = M (B) (cMT)T = c(M)T
(C) (M + N)T = M T + NT (D) MN = NM
Answer: (D)
Exp: Matrix multiplication is not commutative in general.

2. In a housing society, half of the families have a single child per family, while the remaining
half have two children per family. The probability that a child picked at random, has a sibling
is _____
Answer: 0.667
Exp: Let E1 = one children family
E 2 = two children family and
A = picking a child then by Baye’s theorem, required probability is
1

( )
.x
E 2 2
P 2 = = = 0.667
A 1 x 1
. + .x 3
2 2 2
(Here ‘x’ is number of families)

 z2 − z + 4 j 
3. ∫C  z + 2 j 
C is a closed path in the z-plane given by z = 3. The value of the integral → 

dz is
(A) −4π (1 + j2 ) (B) 4 π ( 3 − j2 ) (C) −4π ( 3 + j2 ) (D) 4 π (1 − j2 )
Answer: (C)
Exp: Z = −2 j is a singularity lies inside C : Z = 3
∴ By Cauchy’s integral formula,
Z2 − Z + 4 j
∫ C Z + 2 j dz = 2πj.  Z − Z + 4 j Z=−2 j
2

= 2πj[ −4 + 2 j + 4 j] = −4π [3 + j2]

4. A real (4 × 4) matrix A satisfies the equation A2 = I, where I is the (4 × 4) identity matrix.


The positive eigen value of A is __________.
Answer: 1
1
Exp: A 2 = I ⇒ A = A −1 ⇒ if λ is on eigen value of A then is also its eigen value. Since, we
λ
require positive eigen value. ∴λ = 1 is the only possibility as no other positive number is self
inversed

 India’s No.1 institute for GATE Training  1 Lakh+ Students trained till date  65+ Centers across India
4
EC-GATE-2014 PAPER-01| www.gateforum.com

5. Let X1, X2, and X3 be independent and identically distributed random variables with the
uniform distribution on [0, 1]. The probability P{X1 is the largest} is ________
Answer: 0.32-0.34

6. For maximum power transfer between two cascaded sections of an electrical network, the
relationship between the output impedance Z1 of the first section to the input impedance Z2 of
the second section is
(A) Z2 = Zl (B) Z2 = − Zl (C) Z2 = Z1∗ (D) Z2 = − Z1∗
Answer: (C)
Exp: Two cascaded sections

Section Section
Z1 Z L Z 2
1 2

Z1 = Output impedance of first section


Z2 = Input impedance of second section
For maximum power transfer, upto 1st section is
ZL = Z1*
ZL = Z2 ⇒ Z1*

7. Consider the configuration shown in the figure which is a portion of a larger electrical
network

i5
i2

R R
i3
R
i4
i1 i6

For R = 1Ω and currents i1 = 2A, i4 = -1A, i5 = -4A, which one of the following is TRUE?
(A) i6 = 5 A
(B) i 3 = −4A
(C) Data is sufficient to conclude that the supposed currents are impossible
(D) Data is insufficient to identify the current i 2 ,i3 , and i 6
Answer: (A)

 India’s No.1 institute for GATE Training  1 Lakh+ Students trained till date  65+ Centers across India
5
EC-GATE-2014 PAPER-01| www.gateforum.com

Exp: Given i1 = 2A i5
i 4 = −1A B
i2
i5 = − 4A
1Ω 1Ω
KCL at node A, i1 + i 4 = i 2
⇒ i 2 = 2 − 1 =1A i3
1. KCL at node B, i 2 + i5 = i3 i4
⇒ i3 =1 − 4 = − 3A 1Ω C
A i1 i6
KCL at node C, i3 + i6 = i1
⇒ i6 = 2 − ( −3) = 5A

8. When the optical power incident on a photodiode is 10µW and the responsivity is 0.8 A / W,
the photocurrent generated ( in µA ) is ________.
Answer: 8
Ip
Exp: Responsivity ( R ) =
P0
Ip
0.8 =
10 × 10−6
⇒ I8 = 8µA

9. In the figure, assume that the forward voltage drops of the PN diode D1 and Schottky diode
D2 are 0.7 V and 0.3 V, respectively. If ON denotes conducting state of the diode and OFF
denotes non-conducting state of the diode, then in the circuit,
1kΩ 20 Ω

10 Ω D1 D2

(A) both D1 and D2 are ON (B) D1 is ON and D2 is OFF


(C) both D1 and D2 are OFF (D) D1 is OFF and D2 is ON
Answer: (D) 1K 20Ω
Exp: Assume both the diode ON.
Then circuit will be as per figure (2)
D1 D2
10 − 0.7 10V
∴I = = 9.3mA
1k
0.7 − 0.3 Figure (1)
I D2 = = 20mA
20
1K 20Ω
Now, I D1 = I − ID2 I

= −10.7 mA ( Not possible ) ID1 ID2

∴ D1is OFF and hense D 2 − ON 10V 0.7V 0.3V

 India’s No.1 institute for GATE Training  1 Lakh+ Students trained till date  65+ Centers across India
6
EC-GATE-2014 PAPER-01| www.gateforum.com

10. If fixed positive charges are present in the gate oxide of an n-channel enhancement type
MOSFET, it will lead to
(A) a decrease in the threshold voltage (B) channel length modulation
(C) an increase in substrate leakage current (D) an increase in accumulation capacitance
Answer: (A)

11. A good current buffer has


(A) low input impedance and low output impedance
(B) low input impedance and high output impedance
(C) high input impedance and low output impedance
(D) high input impedance and high output impedance
Answer: (B)
Exp: Ideal current Buffer has Zi = 0
Z0 = ∞

12. In the ac equivalent circuit shown in the figure, if i in is the input current and RF is very large,
the type of feedback is

RD
RD
υout
M2
M1

small signal
RF
input i in

(A) voltage-voltage feedback (B) voltage-current feedback


(C) current-voltage feedback (D) current-current feedback
Answer: (B)
Exp: Output sample is voltage and is added at the input or current
∴ It is voltage – shunt negative feedback i.e, voltage-current negative feedback

13. In the low-pass filter shown in the figure, for a cut-off frequency of 5kHz, the value of R2
( in kΩ ) is ____________.
R2

C
1kΩ 10nF
Vi −
R1 Vo
+

 India’s No.1 institute for GATE Training  1 Lakh+ Students trained till date  65+ Centers across India
7
EC-GATE-2014 PAPER-01| www.gateforum.com

Answer: 3.18
Exp: f = 5KHz
1
Cut off frequency ( LPF ) = = 5KHz
2πR 2 C
1
⇒ R2 = = 3.18 kΩ
2π × 5 × 103 × 10 × 10−9

14. In the following circuit employing pass transistor logic, all NMOS transistors are identical
with a threshold voltage of 1 V. Ignoring the body-effect, the output voltages at P, Q and R
are,

5V 5V 5V
5V

P Q R

(A) 4 V, 3 V, 2 V (B) 5 V, 5 V, 5 V
(C) 4 V, 4 V, 4 V (D) 5 V, 4 V, 3 V
Answer: (C)
5V
Exp: Assume al NMOS are in saturation
∴ VDS ≥ ( VGS − VT )
5V
For m1
M1
(5 − Vp ) ≥ ( 5 − Vp − 1)
(5 − V ) > ( 4 − V ) ⇒ Sat
P
p p
5V
M2
∴ ID1 = k ( VGS − VT )
2

ID1 = K ( 4 − Vp ) ........ (1)


2 Q
5V
M3
For m 2 ,
I D1 = K ( 5 − VQ − 1)
2 For m 3 ,
I D3 = K ( 5 − VR − 1)
2
R
I D2 = K ( 4 − VQ ) ......( 2 )
2

∴ I D 2 = I D3
∴ ID1 = ID2
(4 − V ) = (4 − V )
2 2

(4 − V ) = (4 − V )
2 2 Q R
p Q
⇒ VR = VQ = 4V
⇒ Vp = VQ & Vp + VQ = 8
∴ Vp = VQ = VR = 4V
⇒ Vp = VQ = 4V

 India’s No.1 institute for GATE Training  1 Lakh+ Students trained till date  65+ Centers across India
8
EC-GATE-2014 PAPER-01| www.gateforum.com

15. ( ) ( )
The Boolean expression ( X + Y ) X + Y + X + Y + X simplifies to
(A) X (B) Y (C) XY (D) X+Y
Answer: (A)
Exp: Given Boolean Expression is ( X + Y ) ( X + Y ) + XY + X
As per the transposition theorem
( A + BC ) = ( A + B )( A + C )
so, ( X + Y ) ( X + Y ) = X + YY = X+0

( X + Y ) ( X + Y ) + XY + X ( )
= X + XY .X

= X + ( X + Y ) .X = X + XX. + Y.X = X + 0 + Y.X


Apply absorption theorem = X (1 + Y ) = X.1= X

16. Five JK flip-flops are cascaded to form the circuit shown in Figure. Clock pulses at a
frequency of 1 MHz are applied as shown. The frequency (in kHz) of the waveform at Q3 is
__________ .

1 1 1 J1 Q1 1 J0
J4 Q4 1 J3 Q3 J2 Q2
> clk > clk > clk > clk > clk
1 K2 1 K1
1 K4 1 K2 1 K0

clock

Answer: 62.5
Exp: Given circuit is a Ripple (Asynchrnous) counter. In Ripple counter, o/p frequency of each
flip-flop is half of the input frequency if their all the states are used otherwise o/p frequency
input frequency
of the counter is =
modulus of the counter
input frequency
So, the frequency at Q3 =
16
1×10 6
= H z = 62.5 kHz
16

17. A discrete-time signal x [ n ] = sin ( π2 n ) ,n being an integer,is

(A) periodic with period π . (B) periodic with period π 2 .


(C) periodic with period π / 2 . (D) not periodic
Answer: (D)

 India’s No.1 institute for GATE Training  1 Lakh+ Students trained till date  65+ Centers across India
9
EC-GATE-2014 PAPER-01| www.gateforum.com

Exp: Assume x [ n ] to be periodic, (with period N)


⇒ x [n ] = x [n + N]
⇒ sin ( π 2 n ) = sin ( π2 ( n + N ) )
Every frigonometric function repeate after 2π interval.
⇒ sin ( π2 n + 2πk ) = sin ( π 2 h + π 2 N )
 2k 
⇒ 2πk = π2 N ⇒ N =  
 π 
Since ‘k’ is any integer, there is no possible value of ‘k’ for which ‘N’ can be an integer, thus
non-periodic.

18. Consider two real valued signals, x(t) band-limited to [ −500 Hz, 500 Hz ] and y ( t ) band-
limited to [ −1kHz, 1kHz ] . For z ( t ) = x ( t ) . y ( t ) , the Nyquist sampling frequency (in kHz) is
__________
Answer: 3
Exp: x ( t ) is band limited to [ −500Hz, 500Hz ]
y ( t ) is band limited to [ −1000Hz, 1000Hz ]
z ( t ) = x ( t ) .y ( t )
Multiplication in time domain results convolution in frequency domain.
The range of convolution in frequency domain is [ −1500Hz, 1500 Hz ]
So maximum frequency present in z(t) is 1500Hz Nyquist rate is 3000Hz or 3 kHz

19. A continuous, linear time-invariant filter has an impulse response h(t) described by

h ( t ) = { 30 for 0≤t ≤3
otherwise

When a constant input of value 5 is applied to this filter, the steady state output is _______.
Answer: 45
x (t )
Exp: h (t ) y (t )

y(t) = x (t)* h (t)


x(t) = 5

h (t) =
3

t
3
3
y ( t ) = ∫ 3.5.dτ = 45 ( steady state output )
0

 India’s No.1 institute for GATE Training  1 Lakh+ Students trained till date  65+ Centers across India
10
EC-GATE-2014 PAPER-01| www.gateforum.com

20. The forward path transfer function of a unity negative feedback system is given by
K
G ( s) =
( s + 2 )( s − 1)
The value of K which will place both the poles of the closed-loop system at the same
location, is ______.
Answer: 2.25
K
Exp: Given G ( s ) =
( )( s − 1)
s + 2
H (s) = 1

Characteristic equation: 1 + G ( s ) H ( s ) = 0
K
1+ = 0
( s + 2 )( s − 1)
9
The poles are s1,2 = − 1 ± − 4K
4
9
If − K = 0, then both poles of the closed loop system at the same location.
4
9
So, K = ⇒ 2.25
4

21. Consider the feedback system shown in the figure. The Nyquist plot of G(s) is also shown.
Which one of the following conclusions is correct?

+ Im G ( jω )
k G (s)

−1 +1 Re G ( jω )

(A) G(s) is an all-pass filter


(B) G(s) is a strictly proper transfer function
(C) G(s) is a stable and minimum-phase transfer function
(D) The closed-loop system is unstable for sufficiently large and positive k
Answer: ( D)
Exp: For larger values of K, it will encircle the critical point (-1+j0), which makes closed-loop
system unstable.

 India’s No.1 institute for GATE Training  1 Lakh+ Students trained till date  65+ Centers across India
11
EC-GATE-2014 PAPER-01| www.gateforum.com

22. In a code-division multiple access (CDMA) system with N = 8 chips, the maximum number
of users who can be assigned mutually orthogonal signature sequences is ________
Answer: 7.99 to 8.01
chip rate
Exp: Spreading factor(SF)=
symbol rate
This if a single symbol is represented by a code of 8 chips
Chip rate =80×symbol rate
8 × symbol rate
S.F (Spreading Factor) = =8
symbol rate
Spread factor (or) process gain and determine to a certain extent the upper limit of the total
number of uses supported simultaneously by a station.

23. The capacity of a Binary Symmetric Channel (BSC) with cross-over probability 0.5 is
________
Answer: 0
Exp: Capacity of channel is 1-H(p)
H(p) is entropy function
With cross over probability of 0.5
1 1 1 1
H ( p ) = log 2 + log 2 =1
2 0.5 2 0.5
⇒ Capacity = 1 − 1 = 0

S S12 
24. A two-port network has sattering parameters given by [ S] =  11  . If the port-2 of the
S21 S22 
two-port is short circuited, the S11 parameter for the resultant one-port network is
s11 − s11 s22 + s12s21 s11 − s11 s22 − s12s21
( A) ( B)
1 + s22 1 + s22
s11 − s11 s22 + s12s21 s11 − s11 s22 + s12s21
( C) ( D)
1 − s22 1 − s22
Answer:(B)
Exp:
a1 Two port a2
b1 Network b2

b1 = s11a1 + s12 a 2
b 2 = s 21a1 + s 22 a 2
 b1   s11 s12   a1  b1
 b  = s   ; s1 =
 2   21 s 22  a 2  a1 a 2 =0

By verification Answer B satisfies.

 India’s No.1 institute for GATE Training  1 Lakh+ Students trained till date  65+ Centers across India
12
EC-GATE-2014 PAPER-01| www.gateforum.com

25. The force on a point charge +q kept at a distance d from the surface of an infinite grounded
metal plate in a medium of permittivity ∈ is
q2
(A) 0 (B) away from the plate
16π ∈ d 2
q2 q2
(C) towards the plate (D) towards the plate
16π ∈ d 2 4π ∈ d 2
Answer:(C)
+q
1 Q1Q 2
Exp: F=
4π∈ R 2 d

1 92 92
F= =
4π ∈ ( 2d ) 2 16π∈ d 2
metal plate

Since the charges are opposite polarity d

the force between them is attractive.


−q

Q.No. 26 – 55 Carry Two Marks Each

26. The Taylor series expansion of 3 sin x + 2 cos x is


x3 x3
( A ) 2 + 3x − x 2 − + ....... ( B) 2 − 3x + x 2 − + .......
2 2
x3 x3
( C) 2 + 3x + x 2 + + ....... ( D ) 2 − 3x − x 2 + + .......
2 2
Answer: (A)
 x3   x2 
Exp: 3sin x + 2cos x = 3  x − + ...  + 2 1 − + ... 
 3!   2! 
x3
= 2 + 3x − x 2 − + ...
2

+∞
∫ g ( t ) e − jωt dt = ωe −2 ω for any real value ω . If
2
27. For a Function g(t), it is given that
−∞
+∞
y ( t ) = ∫ g ( τ ) dτ, then ∫ y ( t ) dt is
t

−∞ −∞

j j
(A)0 (B)-j (C) - (D)
2 2
Answer: (B)
Exp: Given

∫ g ( t ).e
− jwt
dt = ω.e−2w ( let G ( jω) )
2

−∞

⇒ ∫ g ( t ) dt = 0
−∞

 India’s No.1 institute for GATE Training  1 Lakh+ Students trained till date  65+ Centers across India
13
EC-GATE-2014 PAPER-01| www.gateforum.com

t
y(t) = ∫ g ( z ).dz ⇒ y ( t ) = g ( t ) * u ( t ) u ( t ) in unit step function 
−∞

⇒ Y ( jω) = G ( jω) .U ( jω)



Y ( jω) = ∫ y ( t ).e
− jω t
dt
−∞

 1 
⇒ Y ( j0 ) = ∫ y ( t ) dt = ω.e
−2w 2
 jω + πδ ( ω)   ω = 0
−∞  
1
= = −j
j

28. The volume under the surface z(x, y) = x + y and above the triangle in the x-y plane defined
by {0 ≤ y ≤ x and 0 ≤ x ≤ 12} is___________.
Answer: 864
12 x

Exp: Volume = ∫∫ Z ( x, y ) dydx = ∫ ∫ ( x + y ) dydx


R x =0 y =0

x 12

12
y2 
12
3 3  x3 
= ∫  xy +  .dx = ∫ x 2 dx =   = 864
x =0 
2 0 0
2 2  3 0

29. Consider the matrix:


0 0 0 0 01
0 0 0 0 10
0 0 0 1 00
J6 =
0 0 10 0 0
0 1 0 0 0 0
1 0 0 0 0 0
Which is obtained by reversing the order of the columns of the identity matrix I 6 .
Let P = I 6 + αJ 6 , where α is a non-negative real number. The value of α for which det(P) =
0 is ___________.
Answer: 1
1 0   0 1 1 α 
Exp: Consider, ( i ) Let P = I 2 + αJ 2 =   + α = 
0 1  1 0   α 1 
⇒ P = 1 − α2
1 0 0 α 
0 1 α 0 
( ii ) Let P = I4 + α J 4 =  
0 α 1 0
 
α 0 0 1

 India’s No.1 institute for GATE Training  1 Lakh+ Students trained till date  65+ Centers across India
14
EC-GATE-2014 PAPER-01| www.gateforum.com

1 α 0 0 1 α
P = (1 ) α 1 0 − (α ) 0 α 1
0 0 1 α 0 0

= (1 − α 2 ) − ( α )  α (1 − α 2 )  = (1 − α 2 )
2

S im ilarly , if P = I 6 + α J 6 th e n w e g et

P = (1 − α 2 )
3

∴ P = 0 ⇒ α = − 1, 1
∵ α is n o n n e g ativ e
∴α =1

30. A Y-network has resistances of 10Ω each in two of its arms, while the third arm has a
resistance of 11Ω in the equivalent ∆ − network, the lowest value ( in Ω ) among the three
resistances is ______________.
Answer: 29.09Ω
Exp:
10Ω 10Ω
X Z

10Ω 11Ω
11Ω
10Ω
Y
Star Connection Delta Connection

X = 29.09Ω
y = 32Ω
(10 )(10 ) + (10 )(11) + (10 )(11)
X= Ω
11
z = 32 Ω
( )( ) ( )(11) + (10 )(11)
10 10 + 10
y= Ω
10
(10 )(10 ) + (10 )(11) + (10 )(11)
z= Ω
10
i.e, lowest value among three resistances is 29.09Ω

31. A 230 V rms source supplies power to two loads connected in parallel. The first load draws
10 kW at 0.8 leading power factor and the second one draws 10 kVA at 0.8 lagging power
factor. The complex power delivered by the source is
(A) (18 + j 1.5) kVA (B) (18 - j 1.5) kVA
(C) (20 + j 1.5) kVA (D) (20 - j 1.5) kVA

 India’s No.1 institute for GATE Training  1 Lakh+ Students trained till date  65+ Centers across India
15
EC-GATE-2014 PAPER-01| www.gateforum.com

Answer: (B)
Exp:
+

L L
o o
230V a a
d d
I II

Load 1:
P = 10 kw 

cos φ = 0.8  SI = P − jQ = 10 − j7.5 KVA
Q = P tan φ = 7.5 KVAR 

Load 2: S = 10 KVA
Q
cos φ = 0.8 sin φ =
S
P
cos φ =
S
P
0.8 = → P = 8kw Q = 6KVAR
10
SI = P + jQ = 8 + j6
Complex power delivered by the source is SI + SII = 18 − j1.5 KVA

32. A periodic variable x is shown in the figure as a function of time. The root-mean-square (rms)
value of x is_______.

x
1

0
t
T/2 T/2

Answer: 0.408

 India’s No.1 institute for GATE Training  1 Lakh+ Students trained till date  65+ Centers across India
16
EC-GATE-2014 PAPER-01| www.gateforum.com

T
1
( x ( t ) ) dt
2
Exp: x rms =
T0∫
2 X
 t 0≤ t ≤ 2
T
x(t) = T
0 T ≤ t ≤T
 2 1

T2 2 T 
1 2 
 .t  .dt + ∫ ( 0 ) .dt 
T  ∫0  T 
=
2
t
 ( 0,0) T T
 T
2  2
T
1 4  t3  2
= .  
T T2  3 0
4 T3 1
x rms = . ⇒ ⇒ 0.408
3T 3 8 6

33. In the circuit shown in the figure, the value of capacitor C(in mF) needed to have critically
damped response i(t) is____________.

40 Ω 4H C
+ −
i (t) VO

Answer: 10mF
Exp: By KVL,
di ( t )
1
v ( t ) = Ri ( t ) + L. i ( t ) dt
C∫
+
dt
Differentiate with respect to time,
R .di ( t ) R di ( ti ) i ( t )
0 = + . + = 0
dt 2 L dt LC
d 2i ( t ) R di ( t ) i ( t )
+ . + = 0
dt 2 L dt LC
−R
2
R 4
±   −
L  L  LC
D1,2 =
2
−R
2
 R  1
D1,2 = ±   −
2L  2L  LC
For critically damped response,
2
 R  1 4L
  = ⇒ C= 2 F
 2L  LC R
Given, L=4H; R= 40Ω
4× 4
C = ⇒ 10mF
( 40 )
2

 India’s No.1 institute for GATE Training  1 Lakh+ Students trained till date  65+ Centers across India
17
EC-GATE-2014 PAPER-01| www.gateforum.com

34. A BJT is biased in forward active mode, Assume VBE = 0.7V, kT / q = 25mV and reverse
saturation current IS = 10−13 A. The transconductance of the BJT (in mA/V) is ________.
Answer: 5.785
KT
Exp: VBE = 0.7V, = 25mV, Is = 10−13
q
IC
Transconductance, g m =
VT
IC = IS  eVBE /VT − 1
= 10−13 e0.7/ 25mV − 1 = 144.625mA
IC 144.625 mA
∴gm = = = 5.785 A / V
VT 25 mV

35. The doping concentrations on the p-side and n-side of a silicon diode are 1 × 1016 cm −3 and
1 × 1017 cm −3 , respectively. A forward bias of 0.3 V is applied to the diode. At T = 300K, the
kT
intrinsic carrier concentration of silicon n i = 1.5 × 1010 cm −3 and = 26mV. The electron
q
concentration at the edge of the depletion region on the p-side is
(A) 2.3 × 109 cm −3 (B) 1 × 1016 cm −3 (C) 1 × 1017 cm −3 (D) 2.25 × 106 cm −3
Answer:(A)
n i 2 Vbi /VT
Exp: Electron concentration, n  e
NA

=
(1.5 × 10 ) 10 2

e0.3/26mV
1 × 1016
= 2.3 × 109 / cm 3

36. A depletion type N-channel MOSFET is biased in its linear region for use as a voltage
controlled resistor. Assume threshold voltage
−8
VTH = 0.5V, VGS = 2.0 V, VDS = 5V, W / L = 100, COX = 10 F / cm and µ n = 800cm / V − s .
2 2

The value of the resistance of the voltage controlled resistor ( in Ω ) is ________.


Answer:500
Exp: Given VT = −0.5V; VGS = 2V; VDS = 5V; W = 100; Cθx = 10−8 f / cm
L
µ n = 800cm 2 / v − s
1 W
I D = µ n C0 x  2 ( VGS − VT ) VDS − VDS2 
2 L 
−1 −1
 ∂I D   ∂ 1 W 
  = rds   µ n C0x  2 ( VGS − VT ) VDS − VDS2   
 ∂VDS   ∂VDS  2 L 

 India’s No.1 institute for GATE Training  1 Lakh+ Students trained till date  65+ Centers across India
18
EC-GATE-2014 PAPER-01| www.gateforum.com

−1
 W W 
= µ n C0 x ( VGS − VT ) − µ n C0x VDS 
 L L 

1
⇒ rds =
W
µ n C0 x ( VGS − VT − VDs )
L
1
= = 500Ω
800 × 10 × 100 ( 2 + 0.5 − 5 )
−8

37. In the voltage regulator circuit shown in the figure, the op-amp is ideal. The BJT has
VBE = 0.7 V and β = 100, and the zener voltage is 4.7V. For a regulated output of 9 V, the
value of R ( in Ω ) is ______ .

VI = 12 V V0 = 9 V

+ 1kΩ
1kΩ

Vz = 4.7 V R

V = 12V 9V
i
Answer:1093
Exp: Given VBE = 0.7V, β = 100, VZ = 4.7V, V0 = 9V + 1K

R
VR = 9 ×
R + 1k VR
R
4.7 = 9 × (∵ VR = Vz ) Vz R
R + 1k
R = 1093 Ω

38. In the circuit shown, the op-amp has finite input impedance, infinite voltage gain and zero
input offset voltage. The output voltage Vout is
R2
(A) − I 2 ( R 1 + R 2 )

(B) I 2 R 2 R1 l1

Vout
(C) I1 R 2 l2 +

(D) − I1 ( R 1 + R 2 )

 India’s No.1 institute for GATE Training  1 Lakh+ Students trained till date  65+ Centers across India
19
EC-GATE-2014 PAPER-01| www.gateforum.com

Answer: (C)
Exp: Given, Zi = ∞
A 0L = ∞
Vi0 = 0 R2

V2 = ( R1 / /R 2 ) I1
R1 I1
R 1R 2
= I1 ...... (1) −
R1 + R 2 V2 V0
+
KCL at inverting node V1

V2 V2 − V0
+ =0 (∴ Zi = ∞ )
R1 R2
V0 1 1 
= V2  + 
R2  R1 R 2 
V0  R 1R 2   R 2 + R 1 
=  I1  
R 2  R 1 + R 2   R 1R 2 
⇒ V0 = I1R 2

39. For the amplifier shown in the figure, the BJT parameters are VBE = 0.7 V, β = 200, and
thermal voltage VT = 25mV. The voltage gain ( v 0 / v i ) of the amplifier is _______.

VCC = +12V

RC
R1 5kΩ
33kΩ vo
1 µF 1 µF
vi

R2 RS
11kΩ 10Ω

R E1 CE
1mF
1k Ω

Answer: -237.76
Exp: VBE = 0.7V, β = 200, VT = 25mV
DC Analysis:
11k
VB = 12 × = 3V
11k + 33k
VE = 3 − 0.7 = 2.3V
2.3
IE = = 2.277 mA
10 + 1k

 India’s No.1 institute for GATE Training  1 Lakh+ Students trained till date  65+ Centers across India
20
EC-GATE-2014 PAPER-01| www.gateforum.com

I B = 11.34 µA
IC = 2.26 mA
25mV
re = = 10.98 Ω
2.277 mA
V −βR C −200 × 5k
AV = 0 = =
Vi β re + (1 + β )( R s ) 200 × 10.98 + ( 201)10
A V = −237.76

40. The output F in the digital logic circuit shown in the figure is

XOR
X
Y AND

Z
XNOR

( A ) F = XYZ + XYZ ( B) F = XYZ + XYZ


( C) F = XYZ + XYZ ( D ) F = XYZ + XYZ
Answer: (A)
Exp: XOR
X
Y

Z
XNOR

Assume dummy variable K as a output of XOR gate K = X ⊕ Y = XY + XY


F = K. ( K  Z )
= ( KZ + K.Z )
= K. KZ + K.K.Z
(
= 0 + K.Z ∵ K. K = 0 and K.K = K )
Put the value of K in above expression
F = ( XY + XY ) Z
= XYZ + XYZ

 India’s No.1 institute for GATE Training  1 Lakh+ Students trained till date  65+ Centers across India
21
EC-GATE-2014 PAPER-01| www.gateforum.com

41. Consider the Boolean function, F ( w, x, y,z ) = wy + xy + wxyz + wxy + xz + xyz. which one
of the following is the complete set of essential prime implicants?
(A) w, y, xz, x z (B) w, y, xz (C) y, x y z (D) y, xz,xz
Answer: (D)
Exp: Given Boolean Function is
F ( w, x, y, z ) = wy + xy + wxyz + wxy + xz + xyz
By using K-map

xz
yz
wx
00 01 11 10

00 1 1 1
01 1 1 1

11 1 1 1

10 1 1 1 y

xz
So, the essential prime implicants (EPI ) are y, xz, xz

42. The digital logic shown in the figure satisfies the given state diagram when Q1 is connected
to input A of the XOR gate.

S=0

S =1
A 00 01
D1 Q1
D2 Q2 S=0
S =1 S =1
> S > S=0
Q1 Q2
CLK
10 11
S =1

S=0

Suppose the XOR gate is replaced by an XNOR gate. Which one of the following options
preserves the state diagram?
(A) Input A is connected to Q 2
(B) Input A is connected to Q2
(C) Input A is connected to Q1 and S is complemented
(D) Input A is connected to Q1
Answer: (D)

 India’s No.1 institute for GATE Training  1 Lakh+ Students trained till date  65+ Centers across India
22
EC-GATE-2014 PAPER-01| www.gateforum.com

Exp: The input of D2 flip-flop is


D 2 = Q1s + Q1 s (∵ A = Q1 )

The alternate expression for EX-NOR gate is = A ⊕ B = A ⊕ B = A ⊕ B


So, if the Ex-OR gate is substituted by Ex-NOR gate then input A should be connected to Q1

D 2 = Q1S + Q1 S = Q1S + Q1 .S (∵ A = Q1 )
= Qi S + Q1 .S

n n
 1   1
43. Lex x [ n ] =   u ( n ) −  −  u ( − n − 1) . The Region of Convergence (ROC) of the z-
 −9   3
transform of x[n]
1 1 1 1
(A) is z > (B) is z < (C) is > z > (D) does not exist.
9 3 3 9
Answer: (C)
 −1   −1 
n n

Exp: Given x [ n ] =   u [ n ] −   u [ −n − 1]
 9   3 
 −1 
h
1
for   u [ n ] R oc in z >
 9  9
(Right sided sequence, R oc in exterior of circle of radius 1 )
9
1 1
Thus overall R oc in <z<
9 3
 πn 
44. Consider a discrete time periodic signal x[n] = sin   . Let a k be the complex Fourier
 s 
series coefficients of x[n]. The coefficients { a k } are non-zero when k = Bm ± 1, where m is
any integer. The value of B is_________.
Answer: 10
 πn 
Exp: Given x [ n ] = sin   ; N = 10
 5 
⇒ Fourier series co-efficients are also periodic with period N = 10
2π 2π
1 j 10 n −1 − i n
x [n] = e e 10
2j 2j
1 −1 −1
a1 = ; a −1 = ⇒ a −1 = a −1+10 = a 9 =
2j 2j 2j
a1 = a1 + 10  a1 = a1 + 20
 or
a −1 = a −1 + 10  a −1 = a −1 + 20
⇒ k = 10 m + 1 or k = 10.m − 1 ⇒ B = 10

 India’s No.1 institute for GATE Training  1 Lakh+ Students trained till date  65+ Centers across India
23
EC-GATE-2014 PAPER-01| www.gateforum.com

45. A system is described by the following differential equation, where u(t) is the input to the
system and y(t) is the output of the system.

y ( t ) + 5y ( t ) = u ( t )
.

When y(0) = 1 and u(t) is a unit step function, y(t) is


(A) 0.2 + 0.8e −5t (B) 0.2 − 0.2e −5t (C) 0.8 + 0.2e −5t (D) 0.8 − 0.8e −5t
Answer: (A)
Exp: Given y ( t ) + 5y ( t ) = u ( t ) and y ( 0 ) =1; u ( t ) is a unit step function.
Apply Laplace transform to the given differential equation.
1
S y ( s) − y ( 0) + 5 y (s ) =
s
1   dy  
y ( s ) [ s + 5] = + y ( 0) L   = s y ( s ) − y ( 0)  L u ( t ) = 1  
s   dt     s 
1
+1
y (s) = s
( s + 5)
( s +1) A B
y (s) = ⇒ +
s ( s + 5) s s+ 5
A= 1 ; B = 4
5 5
1 4
y (s) = +
5s 5 ( s + 5 )
Apply inverse Laplace transform,
1 4 −5t
y(t) = + e
5 5
y ( t ) = 0.2 + 0.8e −5t

46. Consider the state space model of a system, as given below


. 
 x 1   −1 1 0   x1  0   x1 
.    x  + 4  u; y = 1 1 1  x 
 x 2  =  0 −1 0   2   [ ] 2
 .   0 0 −2   x 3   0   x 3 
x 3   
 
The system is
(A) controllable and observable
(B) uncontrollable and observable
(C) uncontrollable and unobservable
(D) controllable and unobservable
Answer: (B)

 India’s No.1 institute for GATE Training  1 Lakh+ Students trained till date  65+ Centers across India
24
EC-GATE-2014 PAPER-01| www.gateforum.com

Exp: From the given state model,


 −1 1 0  0 
A =  0 −1 0  B =  4 
  c = [1 1 1]
 0 0 −2   0 

Controllable: Q c = c =  B AB A 2 B 

if Q c ≠ 0 → controllable

 0 4 −8
Q c =  4 −4 4  ⇒ Qc = 0
 0 0 0 
∴ uncontrollable
 C 
Observable : Q 0 =  CA 
 CA 2 

If Q 0 ≠ 0 → observable

1 1 1
Q 0 =  −1 0 −2  ⇒ Q 0 =1
 1 −1 4 
∴ Observable
The system is uncontrollable and observable

10
47. The phase margin in degrees of G ( s ) = calculated using the
( s + 0.1)( s + 1) + ( s + 10)
asymptotic Bode plot is_______.
Answer: 48
10
Exp: G (s) =
( s + 0.1)( s +1)( s +10 )
10
G (s) =
 s   s
0.1 1 +  [ 1+ s ]  1+  .10
 0.1   10 
10
G (s) =
[1+10s][ 1+ s][ 1+ 0.1s]
10
By Approximation, G ( s ) =
[10s +1]

 India’s No.1 institute for GATE Training  1 Lakh+ Students trained till date  65+ Centers across India
25
EC-GATE-2014 PAPER-01| www.gateforum.com

10
Phase Margin = θ =180 + GH ω=ωgc ωgc = 1 =
100ω2 + 1
 10 × 0.99 
= 180 − tan −1   99
 1  =100ω2 =

Phase Margin = 95° .73
99
⇒ ω2 ⇒ ωgc = 0.9949r / sc

Asymptotic approximation, Phase margin = φ − 45°  48

1
48. For the following feedback system G ( s ) = . The 2% settling time of the step
( s + 1) + ( s + 2 )
response is required to be less than 2 seconds.

r + C (s) G (s)
y

Which one of the following compensators C(s) achieves this?


 1   0.03   s + 8
( A) 3
 s + 5 
( B) 5
 s
+ 1

( C) 2 ( s + 4) ( D) 4
 s + 3
Answer: (C)
Exp: By observing the options, if we place other options, characteristic equation will have 3rd order
one, where we cannot describe the settling time.
If C ( s ) = 2 ( s + 4 ) is considered
The characteristic equation, is
s 2 + 3s + 2 + 2s + 8 = 0
⇒ s 2 + 5s +10 = 0
Standard character equation s 2 + 2 ξωn s + ω2n = 0
ω2n = 10; ξωn = 2.5
4
Given, 2% settling time, < 2 ⇒ ξw n > 2
ξw n
49. Let x be a real-valued random variable with E[X] and E[X2] denoting the mean values of X
and X2, respectively. The relation which always holds true is

( A ) ( E [ X ]) ( B) E  X 2  ≥ ( E [ X ])
2 2
> E  X 2 

( C) E  X 2  = ( E [ X ]) ( D ) E  X 2  > ( E [ X ])
2 2

Answer: (B)
Exp: V ( x ) = E ( x 2 ) − {E ( x )} ≥ 0 i.e., var iance cannot be negative
2

∴ E ( x 2 ) ≥ {E ( x )}
2

 India’s No.1 institute for GATE Training  1 Lakh+ Students trained till date  65+ Centers across India
26
EC-GATE-2014 PAPER-01| www.gateforum.com

50. Consider a random process X ( t ) = 2 sin ( 2 πt + ϕ ) , where the random phase ϕ is uniformly
distributed in the interval [ 0, 2 π] . The auto-correlation E  X ( t1 ) X ( t 2 ) 

( A ) cos ( 2π ( t1 + t 2 ) ) ( B ) sin ( 2π ( t1 − t 2 ) )
( C ) sin ( 2π ( t1 + t 2 ) ) ( D ) cos ( 2π ( t1 − t 2 ) )
Answer: (D)
Exp: Given X(t) = 2 sin ( 2πt + φ )
f φ (θ)
φ in uniformly distributed in the interval [0, 2π ]
1


E [ x(t1 )x(t 2 ) ] = ∫ 2 sin(2 πt1 + θ) 2 sin ( 2πt 2 + θ ) f φ (θ)dθ
0

2π 1
= 2 ∫ sin ( 2πt1 + θ ) sin ( 2πt 2 + θ ).
.dθ
0 2π 0 2π θ
1 2π 1 2π
= ∫
2π 0
sin(2π(t1 + t 2 ) + 2θ)dθ +
2π ∫0
cos(2π(t1 − t 2 )dθ

First integral will result into zero as we are integrating from 0 to 2 π.


Second integral result into cos {2π(t1 − t 2 )}

⇒ E [ X(t1 )X(t 2 )] = cos ( 2π(t1 − t 2 )

51. Let Q ( γ) be the BER of a BPSK system over an AWGN channel with two-sided noise
power spectral density N0/2. The parameter γ is a function of bit energy and noise power
spectral density.
A system with tow independent and identical AWGN channels with noise power spectral
density N0/2 is shown in the figure. The BPSK demodulator receives the sum of outputs of
both the channels.
AWGN
Channel1
0 /1 BPSK BPSK 0 /1
Modulator +
Demodulator
AWGN
Channel 2

( )
If the BER of this system is Q b γ , then the value of b is _____________.
Answer: 1.414
  
 2E    E 
Exp: Bit error rate for BPSK = Q   . Q  
 NO    N O  
  2 
2E
⇒Y=
NO

 India’s No.1 institute for GATE Training  1 Lakh+ Students trained till date  65+ Centers across India
27
EC-GATE-2014 PAPER-01| www.gateforum.com

φ2 ( t )
NO
Function of bit energy and noise PSD
2
Counterllation diagram of BPSK
−a a φ1 ( t )
Channel is A WGN which implies noise sample as independent
Let 2x + n1 + n 2 = x1 + n1

where x1 = 2x
x + n1
+
n = n1 + n 2
1

x 2x + n1 + n2
+
 2E  1
noise in channel 1
Now Bit error rate = Q  
 N O1  x + n2
  +

E1 is energy in x1
noise in channel 2
N O1 is PSD of h1
E1 = 4E [as amplitudes are getting doubled]
N O1 = N O [independent and identical channel]
 4E   2E 
⇒ Bit error rate = Q   = Q 2  ⇒ b = 2 or 1.414
 N   N
 O   O 

52. A fair coin is tossed repeatedly until a ‘Head’ appears for the first time. Let L be the number
of tosses to get this first ‘Head’. The entropy H(L) in bits is _________.
Answer: 2
Exp: In this problem random variable is L
L can be 1, 2,..............

1
P {L = 1} =
2
1
P {L = 2} =
4
1
P {L = 3} =
8
1 1 1 1 1 1 1 1 1
H {L} = log 2 + lg o 2 + log 2 + ......... = 0 + 1. + 2. + 3. + .........
2 1 4 1 8 1 2 4 8
2 4 8
[ Arithmatic gemometric series summation]
1 .1
2 2
= + =2
1− 1
2
 1
2 1 −  
 2

 India’s No.1 institute for GATE Training  1 Lakh+ Students trained till date  65+ Centers across India
28
EC-GATE-2014 PAPER-01| www.gateforum.com

53. In spherical coordinates, let aˆ θ .aˆ φ denote until vectors along the θ, φ directions.
100
E= sin θ cos ( ωt − βr ) aˆ θ V / m and
r
0.265
H= sin θ cos ( ωt − βr ) aˆ φ A / m
r
represent the electric and magnetic field components of the EM wave of large distances r
from a dipole antenna, in free space. The average power (W) crossing the hemispherical shell
located at r = 1km,0 ≤ θ ≤ π / 2 is _______
Answer: 55.5
100
Exp: Eθ = sin θ e − Jβr
r
0.265
HQ = sin θ e− Jβr
r
1
Pavg = ∫ E θ H*Q .ds
2 s
1 100 ( 0.265 ) 2 2
= ∫ sin θ r sin θ dθ dφ
2 s r2
1
Pavt = ∫ ( 26.5 ) sin 2 dθ dφ
2 s
π

( 3 ) ( 2π )
2
=13.25 ∫
θ= 0
sin 3 θdθ ∫
Q =0
dφ = 13.25. 2

P = 55.5 w

54. For a parallel plate transmission line, let v be the speed of propagation and Z be the
characteristic impedance. Neglecting fringe effects, a reduction of the spacing between the
plates by a factor of two results in
(A) halving of v and no change in Z (B) no change in v and halving of Z
(C) no change in both v and Z (D) halving of both v and Z
Answer: (B)
276 d
Exp: Zo = log  
∈r r
d → distance between the two plates
so, zo – changes, if the spacing between the plates changes.
1
V= → independent of spacing between the plates
LC

 India’s No.1 institute for GATE Training  1 Lakh+ Students trained till date  65+ Centers across India
29
EC-GATE-2014 PAPER-01| www.gateforum.com

λ
55. The input impedance of a section of a lossless transmission line of characteristic
8
impedance 50Ω is found to be real when the other end is terminated by a load
Z L ( = R + jX ) Ω. if X is 30 Ω, the value of R ( in Ω ) is _________
Answer: 40
Exp: Given,  = λ
s
Zo = 50Ω

(
Zin  = λ
8 )  Z + JZo 
= Zo  L 
 Zo + KZL 
 Z + J50   ZL + J50 50 − JZL 
Zin = 50  L  = 50  × 
 50 + JZ L   50 + JZL 50 − JZ L 
 50ZL + 50 ZL + J ( 502 − ZL2 ) 
Zin = 50  
 502 + Z2L 
Given , Zin → Re al
So, I mg ( Zin ) = 0
502 − ZL2 = 0
Z2L = 502
R 2 + X 2 = 502
R 2 = 502 − X 2 = 502 − 302
R = 40Ω

 India’s No.1 institute for GATE Training  1 Lakh+ Students trained till date  65+ Centers across India
30
EC-GATE-2014 PAPER-02| www.gateforum.com

Q. No. 1 – 5 Carry One Mark Each

1. Choose the most appropriate word from the options given below to complete the following
sentence.
Communication and interpersonal skills are_____ important in their own ways.
(A) each (B) both (C) all (D) either
Answer: (B)

2. Which of the options given below best completes the following sentence?
She will feel much better if she ________________.
(A) will get some rest (B) gets some rest
(C) will be getting some rest (D) is getting some rest
Answer: (B)

3. Choose the most appropriate pair of words from the options given below to complete the
following sentence.
She could not _____ the thought of _________ the election to her bitter rival.
(A) bear, loosing (B) bare, loosing (C) bear, losing (D) bare, losing
Answer: (C)

4. A regular die has six sides with numbers 1 to 6 marked on its sides. If a very large number of
throws show the following frequencies of occurrence: 1 → 0.167; 2 → 0.167; 3 → 0.152; 4 →
0.166; 5 → 0.168; 6 → 0.180. We call this die
(A) irregular (B) biased (C) Gaussian (D) insufficient
Answer: (B)
Exp: For a very large number of throws, the frequency should be same for unbiased throw. As it
not same, then the die is baised.

5. Fill in the missing number in the series.


2 3 6 15 ___ 157.5 630
Answer: 45
Exp:
2 3 6 15 45 157.5 630

1.5 2 2.5 3 3.5 4

2nd number
is in increa sin g order as shown above
1st number

 India’s No.1 institute for GATE Training  1 Lakh+ Students trained till date  65+ Centers across India
1
EC-GATE-2014 PAPER-02| www.gateforum.com

Q. No. 6 – 10 Carry One Mark Each

6. Find the odd one in the following group


Q,W,Z,B B,H,K,M W,C,G,J M,S,V,X
(A) Q,W,Z,B (B) B,H,K,M (C) W,C,G,J (D) M,S,V,X
Answer: (C)
Exp: a W Z B
B H K N
17 23 26 2
W C G J M S V X
6 3 2 6 4 3 6 3 2
6 3 2

7. Lights of four colors (red, blue, green, yellow) are hung on a ladder. On every step of the
ladder there are two lights. If one of the lights is red, the other light on that step will always
be blue. If one of the lights on a step is green, the other light on that step will always be
yellow. Which of the following statements is not necessarily correct?
(A) The number of red lights is equal to the number of blue lights
(B) The number of green lights is equal to the number of yellow lights
(C) The sum of the red and green lights is equal to the sum of the yellow and blue lights
(D) The sum of the red and blue lights is equal to the sum of the green and yellow lights
Answer: (D)

8. The sum of eight consecutive odd numbers is 656. The average of four consecutive even
numbers is 87. What is the sum of the smallest odd number and second largest even number?
Answer: 163
Exp: Eight consecutive odd number =656
a-6, a-1, a-2, a ,a+2 ,a+4, a+6
a+8=656
a=81
Smallest m=75 … (1)
Average consecutive even numbers
a −2+a +a +2+a +4
⇒ = 87
4
⇒ a = 86
Second largest number =88
1+2=163

9. The total exports and revenues from the exports of a country are given in the two charts
shown below. The pie chart for exports shows the quantity of each item exported as a
percentage of the total quantity of exports. The pie chart for the revenues shows the
percentage of the total revenue generated through export of each item. The total quantity of
exports of all the items is 500 thousand tonnes and the total revenues are 250 crore rupees.
Which item among the following has generated the maximum revenue per kg?

 India’s No.1 institute for GATE Training  1 Lakh+ Students trained till date  65+ Centers across India
2
EC-GATE-2014 PAPER-02| www.gateforum.com

Exports Revenues

Item 6 Item 1
Item 1
Item 6
16% 11%
12%
Item5 Item2 19%
Item 2
12% 20% Item 5
20%
Item3
Item4 20%
19% Item3
22% Item 4
6% 23%

(A) Item 2 (B) Item 3 (C) Item 6 (D) Item 5


Answer: (D)
Exp: Item:2 Item:3
20 23 × 250 × 107
× 250 × 107
100 19 × 500 × 103
20
× 500 × 103
100 1.2 = Item 3
0.5 × 104 = 5 × 103 1 = Item 2
Item: 6 Item:5
19 20 5
= 1.18 = Item 6 = = 1.6 ⇒ 1.6 = Item 5
16 12 3

10. It takes 30 minutes to empty a half-full tank by draining it at a constant rate. It is decided to
simultaneously pump water into the half-full tank while draining it. What is the rate at which
water has to be pumped in so that it gets fully filled in 10 minutes?
(A) 4 times the draining rate (B) 3 times the draining rate
(C) 2.5 times the draining rate (D) 2 times the draining rate
Answer: (A)
Exp: Vhalf = 30(s) drawing rate = s
Total volume =60 S tank
(s1 )(10) − (s)10 = 30s
s1 (s) − s = 3s
s1 = 4s
s1 = 4drawing rate

 India’s No.1 institute for GATE Training  1 Lakh+ Students trained till date  65+ Centers across India
3
EC-GATE-2014 PAPER-02| www.gateforum.com

Q. No. 1 – 25 Carry One Mark Each

1. The determinant of matrix A is 5 and the determinant of matrix B is 40. The determinant of
matrix AB is ________.
Answer: 200
Exp: AB = A . B = ( 5 ) . ( 40 ) = 200

2. Let X be a random variable which is uniformly chosen from the set of positive odd numbers
less than 100. The expectation E[X]is __________.
Answer:50
Exp: X = 1,3,5,....,99 ⇒ n = 50 ( number of observations )
1 n 1 1
∴E(x) = ∑ x i = [1 + 3 + 5 + .... + 99] = ( 50 ) = 50
2

n i =1 50 50

3. For 0 ≤ t < ∞, the maximum value of the function f ( t ) = e − t − 2e −2 t occurs at


(A) t = loge4 (B) t = loge2 (C) t = 0 (D) t = loge8
Answer: (A)
Exp: f ' ( t ) = −e − t + 4e−2t = 0
1
⇒ e − t  4e − t − 1 ⇒ e− t = ⇒ t = log e4
4
and f ( t ) < 0 at t = log e
'' 4

x
 1
4. The value of lim  1 +  is
x →∞  x
(A) ln2 (B) 1.0 (C) e (D) ∞
Answer: (C)
x
 1
Exp: lim  1 +  = e ( standard limit )
x →∞
 x

5. If the characteristic equation of the differential equation


d2y dy
2
+ 2α +y=0
dx dx
has two equal roots, then the values of α are
( A) ± 1 ( B) 0,0 ( C) ± j ( D ) ± 1/ 2
Answer: (A)
Exp: For equal roots, Discriminant B2 − 4AC = 0
⇒ 4α 2 − 4 = 0
⇒ α = ±1

 India’s No.1 institute for GATE Training  1 Lakh+ Students trained till date  65+ Centers across India
4
EC-GATE-2014 PAPER-02| www.gateforum.com

6. Norton’s theorem states that a complex network connected to a load can be replaced with an
equivalent impedance
(A) in series with a current source (B) in parallel with a voltage source
(C) in series with a voltage source (D) in parallel with a current source
Answer: (D)
Exp: Norton’s theorem

IN Zequ Load

7. In the figure shown, the ideal switch has been open for a long time. If it is closed at t=0, then
the magnitude of the current (in mA) through the 4 kΩ resistor at t = 0+ is _______.
5kΩ 4 kΩ 1kΩ
i
10 V −+ 10µF 1mH
t=0

5 kΩ 4 kΩ
Answer: 1.2 mA
Exp: For t = o+
10V + •
10 − •
i (o +) = ⇒ 1.11mA i (0+)
9K
i ( o + )  1.2 mA

8. A silicon bar is doped with donor impurities ND = 2.25 x 1015 atoms / cm3. Given the intrinsic
carrier concentration of silicon at T = 300 K is ni = 1.5 x 1010 cm-3. Assuming complete
impurity ionization, the equilibrium electron and hole concentrations are
(A) n0 = 1.5 x 1016 cm-3, p0 = 1.5 x 105 cm-3
(B) n0 = 1.5 x 1010 cm-3, p0= 1.5 x 1015 cm-3
(C) n0 = 2.25 x 1015 cm-3, p0 = 1.5 x 1010 cm-3
(D) n0 = 2.25 x 1015 cm-3, p0 = 1 x 105 cm-3
Answer: (D)
N D = 2.25 × 1015 Atom / cm3
Exp:
h i = 1.5 × 1010 / cm 3
Since complete ionization taken place,
h 0 = N D = 2.25 × 1015 / cm3

n i 2 (1.5 × 10 )
10 2

P0 = = = 1 × 105 / cm 3
n0 2.25 × 1015

 India’s No.1 institute for GATE Training  1 Lakh+ Students trained till date  65+ Centers across India
5
EC-GATE-2014 PAPER-02| www.gateforum.com

9. An increase in the base recombination of a BJT will increase


(A) the common emitter dc current gain β
(B) the breakdown voltage BVCEO
(C) the unity-gain cut-off frequency fT
(D) the transconductance gm
Answer: (B)

10. In CMOS technology, shallow P-well or N-well regions can be formed using
(A) low pressure chemical vapour deposition
(B) low energy sputtering
(C) low temperature dry oxidation
(D) low energy ion-implantation
Answer: (D)

11. The feedback topology in the amplifier circuit (the base bias circuit is not shown for
simplicity) in the figure is VCC

Rc Io
(A) Voltage shunt feedback
Vo

(B) Current series feedback


RS
(C) Current shunt feedback
RE
VS ~
(D) Voltage series feedback
Answer: (B)
Exp: By opening the output feed back signed becomes zero. Hence it is current sampling.
As the feedback signal vf is subtracted from the signal same vs it is series mixing.
12. In the differential amplifier shown in the figure, the magnitudes of the common-mode and
differential-mode gains are Acm and Ad, respectively. If the resistance RE is increased, then
(A) Acm increases
VCC
(B) common-mode rejection ratio increases
RC RC
(C) Ad increases
(D) common-mode rejection ratio decreases + V0 −
Answer: (B)
Exp: A d does not depend on R E +
Vi
A cm decreases as R E is increased −

Ad RE
∴ CMRR = = Increases I0
A cm
− VEE

 India’s No.1 institute for GATE Training  1 Lakh+ Students trained till date  65+ Centers across India
6
EC-GATE-2014 PAPER-02| www.gateforum.com

13. A cascade connection of two voltage amplifiers A1 and A2 is shown in the figure. The open-
loop gain Av0, input resistance Rin, and output resistance RO for A1 and A2 are as follows:
A1:A v 0 = 10, R in = 10kΩ, R 0 = 1kΩ
A2 : A v0 = 5, R in = 5kΩ, R 0 = 200 Ω
The approximate overall voltage gain Vout / Vin is __________.

+ +

Vin A1 A2 R L Vout
1kΩ
− −

Answer: 34.722
V0  Zi 2   R L 
Exp: Overall voltage gain, A v = = A V1 A V2   
Vi  Zi2 + Z01   R L + Z02 
 5k   1k 
= 10 × 5   
 5k + 1k  1k + 200 
A V = 34.722

14. For an n-variable Boolean function, the maximum number of prime implicants is
(A) 2(n-1) (B) n/2 (C) 2n (D) 2(n-1)
Answer: (D)
Exp: For an n-variable Boolean function, the maximum number of prime implicants = 2 ( n −1)

15. The number of bytes required to represent the decimal number 1856357 in packed BCD
(Binary Coded Decimal) form is __________ .
Answer: 4
Exp: In packed BCD (Binary Coded Decimal) typically encoded two decimal digits within a single
byte by taking advantage of the fact that four bits are enough to represent the range 0 to 9.
So, 1856357 is required 4-bytes to stored these BCD digits

16. In a half-subtractor circuit with X and Y as inputs, the Borrow (M) and Difference (N = X - Y)
are given by
(A) M = X, ⊕ Y, N = XY (B) M = XY, N = X⊕ Y
(C) M = X Y , ⊕ N = X ⊕ Y (D) M = XY N = X⊕Y
Answer: (C)

 India’s No.1 institute for GATE Training  1 Lakh+ Students trained till date  65+ Centers across India
7
EC-GATE-2014 PAPER-02| www.gateforum.com

Exp: Function Table for Half-subtractor is

X Y Difference (N) Borrow (M)


0 0 0 0
Hence, N = X ⊕ Y and m = XY
0 1 1 1
1 0 1 0
1 1 0 0

Hence, N = X ⊕ Y and m = XY

17. An FIR system is described by the system function


7 3
H ( z ) = 1 + z −1 + z −2 The system is
2 2
(A) maximum phase (B) minimum phase (C) mixed phase (D) zero phase
Answer: (C)
Exp: Minimum phase system has all zeros inside unit circle maximum phase system has all zeros
outside unit circle mixed phase system has some zero outside unit circle and some zeros
inside unit circle.
7 3
for H ( s ) = 1 + z −1 + z −2
2 2
One zero is inside and one zero outside unit circle hence mixed phase system

18. Let x[n] = x[-n]. Let X(z) be the z-transform of x[n]. If 0.5 + j 0.25 is a zero o X(z), which
one of the following must also be a zero of X(z).
(A) 0.5 - j0.25 (B) 1/(0.5 + j0.25)
(C) 1/(0.5 - j0.25) (D) 2 + j4
Answer: (B)
Exp: Given x [ n ] = x [ −n ]
⇒ x ( z ) = x ( z −1 ) [ Time reversal property in z − transform ]
⇒ if one zero is 0.5 + j0.25
1
then other zero will be
0.5 + j0.25

19. Consider the periodic square wave in the figure shown.


x
1

0 1 2 3 4 t

−1

 India’s No.1 institute for GATE Training  1 Lakh+ Students trained till date  65+ Centers across India
8
EC-GATE-2014 PAPER-02| www.gateforum.com

The ratio of the power in the 7th harmonic to the power in the 5th harmonic for this waveform
is closest in value to _______.
Answer: 0.5
1
Exp: For a periodic sequence wave, nth harmonic component is α
n
1
⇒ power in nth harmonic component is α 2
n
⇒ Ratio of the power in 7 harmonic to power in 5th harmonic for given waveform is
th

1 2
7 = 25 ≈ 0.5
1 2 49
5

20. The natural frequency of an undamped second-order system is 40 rad/s. If the system is
damped with a damping ratio 0.3, the damped natural frequency in rad/s is ________.
Answer: 38.15 r / sec
Exp: Given ωn = 40 r / sec
ξ = 0.3
ωd = ωn 1 − ξ 2

ωd = 40 1 − ( 0.3)
2

ωd = 38.15 r / sec

21. For the following sytem,


x 2 (s)
+
x1 ( s ) + s + 1 Y (s)
s +1
− s

y ( s)
When X1 ( s ) = 0 , the transfer function is
x 2 ( s)
s +1 1 s+2 s +1
( A) ( B) ( C) ( D)
s2 s +1 s ( s + 1) s ( s + 2)
Answer: (D)
Exp: If X1 ( s ) = 0
+
Y (s) X2 ( s )
1 Y (s)
; The block diagram becomes s
X2 (s ) −

1 1
Y (s) s s ( s + 1) +S
= = ⇒
X2 (s) 1 s (s + 2) / s + 1 s (s + 2) ( S + 1)
1+ .
s ( s + 1)

 India’s No.1 institute for GATE Training  1 Lakh+ Students trained till date  65+ Centers across India
9
EC-GATE-2014 PAPER-02| www.gateforum.com

22. The capacity of a band-limited additive white Gaussian noise (AWGN) channel is given by
 P 
C = W log 2  1 + 2  bits per second (bps), where W is the channel bandwidth, P is the
 σ w
average power received and σ2 is the one-sided power spectral density of the AWGN. For a
P
fixed 2 = 1000, , the channel capacity (in kbps) with infinite bandwidth ( W → ∞ ) is
σ
approximately
(A) 1.44 (B) 1.08 (C) 0.72 (D) 0.36
Answer: (A)
 P 
ω ln 1 + 2 
Exp:
 P 
C = lim ω log 2 1 + 2  = lim  σ ω
w →∞
 σ ω ω→∞ ln 2

 P   P 
ln 1 + 2  ln 1 + 2 
 σ ω . σ ω
lim 
1 P P
= lim = 2
ln 2 ω→∞ P σ 2
σ ln 2 ω→∞ P
σω2
σ2 ω 


This lim it is equivalent to

ln [1 + x ] P P
lim =1 = = ln 2 e 2 = 1.44 KGpa
ω→∞ x σ .ln 2
2
σ

23. Consider sinusoidal modulation in an AM system. Assuming no overmodulation, the


modulation index ( µ ) when the maximum and minimum values of the envelope,
respectively, are 3 V and 1 V, is ________.
Answer: 0.5
A ( t )max − A ( t ) min
Exp: µ=
A ( t ) max + A ( t ) min
3 −1 1
µ= = = 0.5
3 +1 2

24. To maximize power transfer, a lossless transmission line is to be matched to a resistive load
impedance via a λ / 4 transformer as shown.

lossless transmission line


λ / 4 transformer

Z L = 50Ω Z L = 100Ω

The characteristic impedance ( in Ω ) of the λ / 4 transformer is _________.


Answer: 70.7Ω

 India’s No.1 institute for GATE Training  1 Lakh+ Students trained till date  65+ Centers across India
10
EC-GATE-2014 PAPER-02| www.gateforum.com

Exp: Here impedance is matched by using QWT λ ( 4)


∴ Z'0 = Z L Zin
= 100 × 50 = 50 2
= Z'0 = 70.7Ω
25. Which one of the following field patterns represents a TEM wave travelling in the positive x
direction?
( A ) E = +8y,ˆ H = −4zˆ ( B) E = −2y,
ˆ H = −3zˆ

( C) E + 2z,ˆ H = +2yˆ ( D) E = −3y,


ˆ H = +4zˆ
Answer: (B)
Exp: For TEM wave
Electric field (E), Magnetic field (H) and
Direction of propagation (P) are orthogonal to each other.
Here P = + a x
By verification
E = − 2a y , H = − 3a z
E × H = − a y ×− a z = + a x → P

Q. No. 26 – 55 Carry Two Marks Each

26. The system of linear equations


 2 1 3  a   5 
 3 0 1  b =  −4 has
    
 1 2 5  c   14 
(A) a unique solution (B) infinitely many solutions
(C) no solution (D) exactly two solutions
Answer: (B)
2 1 3 5 
Exp: [ A / B] = 3 0 1 −4
1 2 5 14 

2 1 3 5  2 1 3 5 
R 2 → 2R 2 − 3R 1   R3 +R 2  
 0 − 3 − 7 −23 → 0 − 3 − 7 −23
R 3 → 2R 3 − R 1
0 3 7 23  0 0 0 0 
Since, rank ( A ) = rank ( A / B ) < number of unknowns
∴ Equations have infinitely many solutions.

 India’s No.1 institute for GATE Training  1 Lakh+ Students trained till date  65+ Centers across India
11
EC-GATE-2014 PAPER-02| www.gateforum.com

27. The real part of an analytic function f(z) where z =.x + jy is given by e-y cos(x). The
imaginary part of f(z) is
(A) eycos(x) (B) e-ysin(x) (C) -eysin(x) (D) –e-ysin(x)
Answer: (B)
Exp: real part u = e− y cos x and V = ?
∂v ∂v
dv = dx + dy
∂x ∂y
∂u ∂u
= − dx + dy ( U sin g C − R equations ) = e − y cos xdx − e − y sin xdy = d  e − y sin x 
∂y ∂x
Integrating, we get V = e − y sin x

28. The maximum value of the determinant among all 2×2 real symmetric matrices with trace 14
is ________.
Answer: 49
y x
Exp: General 2 × 2 real symmetric matrix is  
x z 
⇒ det = yz − x 2 and trace is y + z = 14 ( given )
⇒ z = 14 − y .............. ( *)
Let f = yz − x 2 ( det ) = − x 2 − y 2 + 14y ( u sin g *)
Using maxima and minima of a function of two variables, we have f is maximum at
x = 0, y = 7 and therefore, maximum value of the determinant is 49


29. If r = xaˆ x + yaˆ y + zaˆ z and r = r, then div ( r 2 ∇ ( ln r ) ) = _______.

Answer: 3

∇ ( ln r ) = 2 ⇒ div ( r 2∇ ( ln r ) ) = div ( r ) = 3
r 
Exp:
r

 ∂  1  x  1 r
∇ ( ln r ) = ∑ aˆ x ∂x ( ln r ) = ∑ aˆ x  r  r  = r 2 ∑ aˆ x x = r 2 
   

30. A series LCR circuit is operated at a frequency different from its resonant frequency. The
operating frequency is such that the current leads the supply voltage. The magnitude of
current is half the value at resonance. If the values of L, C and R are 1 H, 1 F and 1Ω ,
respectively, the operating angular frequency (in rad/s) is ________.
Answer: 0.45 r/sec
Exp: The operating frequency (wx), at which current leads the supply.
i.e., ωx <ωr
again magnitude of current is half the value at resonance

 India’s No.1 institute for GATE Training  1 Lakh+ Students trained till date  65+ Centers across India
12
EC-GATE-2014 PAPER-02| www.gateforum.com

V
i,e.,. at ω = ωx ⇒ I x =
z
V
at ω= ωx ⇒ Iresonance =
R
I resonance
Ix =
2
V V
i.e., = = Z = 2R
z 2R
Given R = 1Ω; L=1H; C=1F
2
 1 
Z = R2 + − ωL  = 2
 ωc 
2
 1 
=R + 
2
− ωL  = 4
 ωc 
By substituting R, L & C values,
2
1  1
⇒ 1 +  − ω  = 4 ⇒ ω2 = 2 = 5
ω  ω
1
Assume ω2 = x, then, x + = 5
x
⇒ x 2 − 5x + 1= 0
x1, 2 = 4.791, 0.208
if x = 4.791 ⇒ ω= 2.18 r sec
if x = 0.208 ⇒ ω= 0.45r sec
But ωx < ωr
So, operating frequency ωx = 0.45 r sec

31. In the h-parameter model of the 2-port network given in the figure shown, the value of h22 (in
S) is _____ .

3Ω

3 Ω 3Ω
1 2

2Ω
1' 2'
2Ω 2Ω

Answer: 1.24
Exp: If two, π − n ws are connected in parallel,
The y-parameter are added

 India’s No.1 institute for GATE Training  1 Lakh+ Students trained till date  65+ Centers across India
13
EC-GATE-2014 PAPER-02| www.gateforum.com

i.e., y equ = y1 + y 2
 2 −1   1 −1 
 3 3  2
y1 = y2 =
− 1 2   −1 1 
 3 3   2 
5 −5 
3 6
y equ = 
 −5 5 
 6 3
 1 − y12 
 y11 y11 
h =  
 y 21 ∆y 
 y11 y11 
where ∆y = y11 y 22 − y12 − y 21
 5  5     −5  −5  
The value of h 22 = ∆y =     −     
 3  3     6  6  
∆y = 2.0833
y11 = 5 ∴ h 22 = 1.24
3

32. In the figure shown, the capacitor is initially uncharged. Which one of the following
expressions describes the current I(t) (in mA) for t > 0?
R1

1kΩ
I
5V +−
C
R2 1 µF
2 kΩ

( A) I ( t ) =
5
3
( ) 2
1 − e − t / τ , τ = msec
3
( B) I ( t ) =
5
2
( ) 2
1 − e − t / τ , τ = msec
3

( C) I ( t ) =
5
2
( )
1 − e − t / τ , τ = 3msec ( D) I ( t ) =
5
2
( )
1 − e − t / τ , τ = 3 msec

Answer: (A)
−t
Exp: ν c ( t ) = VR 2 ( t ) = Vfinal + [ Vinitial − Vfinal ] e τ

2
τ = R equ .Cequ ⇒ ×103 ×10−6
3
2
R equ = 2K  1K ⇒ KΩ
3
Cequ = 1µF

 India’s No.1 institute for GATE Training  1 Lakh+ Students trained till date  65+ Centers across India
14
EC-GATE-2014 PAPER-02| www.gateforum.com

2
τ = msec
3
Vinitial = 0volts
2 10
Vfinal = Vs.s = 5. = volts
3 3
10 10 − t τ
νR 2 ( t ) = − e
3 3
10  −t ν (t) 5 −t
νR 2 ( t ) = 1 − e τ  volts ⇒ i R 2 (t) = R 2 =  1 − e τ  mA
3 
 2K 3 

33. In the magnetically coupled circuit shown in the figure, 56 % of the total flux emanating from
one coil links the other coil. The value of the mutual inductance (in H) is ______ .
M
10 Ω

4H 5H (1/16) F
(
60 cos 4t + 300 V ~ )
Answer: 2.49 Henry
Exp: Given 56% of the total flux emanating from one coil links to other coil.
i.e, K = 56% ⇒ 0.56
M
We have, K =
L1L 2
L1 = 4H; L 2 = 5H
M = ( 0.56 ) 20 ⇒ m = 2.50H

34. Assume electronic charge q = 1.6×10-19 C, kT/q = 25 mV and electron mobility µ n = 1000
cm2/V-s. If the concentration gradient of electrons injected into a P-type silicon sample is
1×1021/cm4, the magnitude of electron diffusion current density (in A/cm2) is _________.
Answer: 4000
kJ
Exp: Given q = 1.6 × 10−19 ; = 2.5 mV, µ n = 1000cm 2 / v − s
q
D n kJ
From Einstein relation, =
µn q
⇒ D n = 25mV × 1000cm / v − S
2

⇒ 25cm 2 / s
dn
Diffuion current Density J = q D n
dx
= 1.6 × 10−19 × 25 × 1 × 1021
= 4000 A / cm 2

 India’s No.1 institute for GATE Training  1 Lakh+ Students trained till date  65+ Centers across India
15
EC-GATE-2014 PAPER-02| www.gateforum.com

35. Consider an abrupt PN junction (at T = 300 K) shown in the figure. The depletion region
width Xn on the N-side of the junction is 0.2 µm and the permittivity of silicon ( ε si ) is
1.044×10-12 F/cm. At the junction, the approximate value of the peak electric field (in kV/cm)
is _________.

P + − region N − region
X11
N A >> N D N D = 1016 / cm3

Answer: 30.66
Exp: Given x n = 0.2 µm, ∈Si = 1.044 × 10−12 F / µ n
N D = 1016 / cm3
q ND xn
Peak Electric field, E =

1.6 × 10−19 × 1016 × 0.00002
= = 30.66 KV / cm
1.044 × 10−12

36. When a silicon diode having a doping concentration of NA = 9 × 1016 cm-3 on p-side and ND =
1 × 1016 cm-3 on n-side is reverse biased, the total depletion width is found to be 3µm . Given
that the permittivity of silicon is 1.04 × 10–12 F/cm, the depletion width on the p-side and the
maximum electric field in the depletion region, respectively, are
(A) 2.7 µm and 2.3 × 105 V/cm (B) 0.3 µm and 4.15 × 105 V/cm
(C) 0.3 µm and 0.42 × 105 V/cm (D) 2.1 µm and 0.42 × 105 V/cm
Answer: (B)
Exp: Given N A = 9 × 1016 / cm3 ; N D = 1 × 1016 / cm3
Total depletion width x = x n + x p = 3 µm
∈= 1.04 × 10−12 F / cm
x n N A 9 × 1016
= =
x p N D 1 × 1016
x n = 9x p ......... (1)
Total Depletion width, x n + x p = 3µm
9x p + x p = 3 µm
x p = 0.3 µm
qN A x p 1.6 × 10−19 × 9 × 1016 × 0.3 µm
Max. Electric field, E = =
∈ 1.04 × 10−12
= 4.15 × 105 V / cm

 India’s No.1 institute for GATE Training  1 Lakh+ Students trained till date  65+ Centers across India
16
EC-GATE-2014 PAPER-02| www.gateforum.com

37. The diode in the circuit shown has Von = 0.7 Volts but is ideal otherwise.
If Vi = 5sin ( ωt ) Volts, the minimum and maximum values of VO (in Volts) are, respectively,

1kΩ
Vi Vo
R1
R2
1kΩ

+
− 2V

(A) -5 and 2.7 (B) 2.7 and 5 (C) -5 and 3.85 (D) 1.3 and 5
Answer: (C)
Exp: When Vi makes Diode 'D' OFF, V0 = Vi
∴ V0 ( min ) = −5V

When Vi makes diode 'D ' ON,

V0 =
( Vi − 0.7 − 2 ) + V + 2V
R1 + R 2
on

∴ V0 ( max ) =
( 5 − 0.7 − 2 )1k + 0.7 + 2V
1k + 1k
= 3.85V

38. For the n-channel MOS transistor shown in the figure, the threshold voltage VTh is 0.8 V.
Neglect channel length modulation effects. When the drain voltage VD = 1.6 V, the drain
current ID was found to be 0.5 mA. If VD is adjusted to be 2 V by changing the values of R
and VDD, the new value of ID (in mA) is

VDD

G
S

(A) 0.625 (B) 0.75 (C) 1.125 (D) 1.5


Answer: (C)

 India’s No.1 institute for GATE Training  1 Lakh+ Students trained till date  65+ Centers across India
17
EC-GATE-2014 PAPER-02| www.gateforum.com

Exp: Given VTh = 0.8V


1 w
When VD = 1.6V, ID = 0.5mA = µ n cos ( VDS − VTh )
2

2 L
[∵ Device is in sat ]
1 ω
⇒ µ n cos = 0.78125 × 10−3 A / V 2
2 L
When VD = 2V
1 ω
I D = µ n cos ( VDS − VTh )
2

2 L
= 078125 × 10−3 ( 2 − 0.8 )1.125mA

39. For the MOSFETs shown in the figure, the threshold voltage Vt = 2V and
1  W
K= µC∞   = 0.1 mA / V 2 . The value of ID (in mA) is ________.
2  L

VDD = +12 V

R1
10 kΩ

R2
ID
10 kΩ

VDD = −5V

Answer: 0.9
1 W 12V
Exp: Given Vt = 2V, K = µ cos = 0.1A / V 2
2 L

( )
1 W 2 10 K
I D1 = I D2 = µ n cos VGs1 − Vt
2 L
= 0.1mA / V 2 ( 5 − 2 )
2
10 K
ID
= 0.9 mA

−5V

 India’s No.1 institute for GATE Training  1 Lakh+ Students trained till date  65+ Centers across India
18
EC-GATE-2014 PAPER-02| www.gateforum.com

40. In the circuit shown, choose the correct timing diagram of the output (y) from the given
waveforms W1, W2, W3 and W4.

X1 D Q
FF1
Clk > Q
output ( y )

X2 D Q

> Q

ClK

X1

X2

W1

W2

W3

W4

(A) W1 (B) W2 (C) W3 (D) W4


Answer: (C)
Exp: This circuit has used negative edge triggered, so output of the D-flip flop will changed only
when CLK signal is going from HIGH to LOW (1 to 0)
1 1 1 1

CLK 0 0 0 0

X1

X2

Y( w 3 )

 India’s No.1 institute for GATE Training  1 Lakh+ Students trained till date  65+ Centers across India
19
EC-GATE-2014 PAPER-02| www.gateforum.com

This is a synchronous circuit, so both the flip flops will trigger at the same time and will
respond on falling edge of the Clock. So, the correct output (Y) waveform is associated to w3
waveform.

41. The outputs of the two flip-flops Q1, Q2 in the figure shown are initialized to 0,0. The
sequence generated at Q1 upon application of clock signal is

Q1
J1 Q1 J2 Q2
> >
K1 Q1 K2Q2

CLK

(A) 01110… (B) 01010… (C) 00110… (D) 01100…


Answer: (D)
Exp:

J1 ( Q 2 ) K1 ( Q 2 ) J 2 ( Q1 ) K 2 ( Q1 ) Q1 Q2
Clock
Initial → - - - - 0 0
1st CP → 1 0 0 1 1 0
2nd CP → 1 0 1 0 1 1
3rd CP →
0 1 1 0 0 1
4th CP→
0 1 0 1 0 0
So, the output sequence generated at Q1 is 01100….

42. For the 8085 microprocessor, the interfacing circuit to input 8-bit digital data (DI0 – DI7)
from an external device is shown in the figure. The instruction for correct data transfer is
3 − to − 8 7
Decoder 6 I / O Device
A2 C 5
(A) MVI A, F8H 4 Data Bus
A1 B Digital
3 DI 0 − DI 7 DO0 − D7 ( D − D )
0 7
A0 A 2 Inputs
1
0
(B) IN F8H G 2A G 2B G1 DS1 DS2
10 / M
RD
A3 A8
A
(C) OUT F8H A5 4 A9
A6 A10
A7 A11
A12 A
13
A14
(D) LDA F8F8H A15

 India’s No.1 institute for GATE Training  1 Lakh+ Students trained till date  65+ Centers across India
20
EC-GATE-2014 PAPER-02| www.gateforum.com

Answer: (D)
Exp: This circuit diagram indicating that it is memory mapped I/O because to enable the 3-to-8
decoder G 2A is required active low signal through ( Io m ) and G 2B is required active
low through (R )
D it means I/o device read the status of device LDA instruction is
appropriate with device address.
Again to enable the decoder o/p of AND gate must be 1 and Ds 2 signal required is 1 which is
the o/p of multi-i/p AND gate to enable I/O device.
So,
A15 A14 A13 A12 A11 A10 A 9 A 8 A 7 A 6 A 5 A 4 A 3 A 2 A1 A 0
1 1 1 1 1 0 0 0 1 1 1 1 1 0 0 0
   
F 8 F 8
Device address = F8F8H
The correct instruction used → LDA F8F8H

43. Consider a discrete-time signal


n for 0 ≤ n ≤ 10
x [n] = 
0 otherwise
If y[n] is the convolution of x[n] with itself, the value of y[4] is _________.
Answer: 10
 n for 0 ≤ n ≤ 10 
Exp: Given x [ n ] =  
0 elsewhere 

y [n ] = x [n ] * x [n ]
n
y [ n ] = ∑ x [ k ].x [ n − k ]
k =0

4
⇒ y [ 4] = ∑ x [ k ].x [ G − k ]
k =0

= x ( 0 ) .x ( 4 ) + x (1) x ( 3) + x ( 2 ) x ( 2 ) + x ( 3) x (1) + x ( 4 ) .x ( 0 )
= 0 + 3 + 4 + 3 + 0 = 10

44. The input-output relationship of a causal stable LTI system is given as


y [ n ] = αy [ n − 1] + βx [ n ]

If the impulse response h[n] of this system satisfies the condition Σ ∞n =0 h [ n ] = 2, the
relationship between α and β is

( A) α = 1− β/ 2 ( B) α = 1+ β / 2 ( C) α = 2β ( D) α = −2β

 India’s No.1 institute for GATE Training  1 Lakh+ Students trained till date  65+ Centers across India
21
EC-GATE-2014 PAPER-02| www.gateforum.com

Answer: (A)
Exp: Given system equation as
y [ n ] = α y [ n − 1] + βx [ n ]

y (z) β
⇒ =
x (z) 1 − α z −1

β
⇒ H (z) =
1 − α z −1

h [n] = β (α ) u [n] [ causal system ]


h


Also given that ∑ h [n] = 2
h =0

 1 
β =2
1 − α 
β
1− α =
2
β
α =1−
2


45. The value of the integral ∫ sin c 2 ( 5t ) dt is ____________.
−∞

Answer: 0.2
Exp: We can use pasrevalis theorem
sin 5πt
Let x ( t ) sin ( 5t ) =
5πt
⇒ in frequency domain
1 X (f )
5

−2.5 −2.5 f

∞ ∞ 2.5 2
1
Now, ∫ x ( t ) dt = ∫ x ( t ) df = ∫  
2 2

−2.5  
−∞ −∞
5

1 1
= × 5 = = 0.2
25 5

 India’s No.1 institute for GATE Training  1 Lakh+ Students trained till date  65+ Centers across India
22
EC-GATE-2014 PAPER-02| www.gateforum.com

46. An unforced liner time invariant (LTI) system is represented by

 x 1   −1 0   x1 
 =    
 x 2   0 −2   x 2 
If the initial conditions are x1(0) = 1 and x2(0) = -1, the solution of the state equation is

( A) x1 ( t ) = −1, x 2 ( t ) = 2 ( B) x1 ( t ) = −e − t , x 2 ( t ) = 2e − t

(C) x1 ( t ) = e − t , x 2 ( t ) = −e−2t ( D) x1 ( t ) = −e − t , x 2 ( t ) = −2e − t

Answer: (C)

Exp: Solution of state equation of X ( t ) = L−1 SI − A −1  .X ( 0 )

 1  −1 0 
X ( 0) =   A =  
 −1  0 − 2

−1
S + 1 0 
[SI − A ]
−1
=
 0 S + 2 

1 S + 2 0
=
(S + 1)(S + 2 )  0 S + 1

 1 
 0 
[SI − A ] =  S + 1
−1

 0 1 
 S + 2 

 −1  1  
L  S + 1 0 
 
L ( SI − A )  = 
−1
−1  
   −1  1 

 0 L   
 S + 2 

e− t 0 
L−1 ( SI − A )  = 
−1
  0 
 e−2t 

 X1 ( t )  e − t 0   1
 =  
 X 2 ( t )   0 e −2t   −1

 X1 ( t )   −e t  X1 ( t ) = e− t
  =  −2t  ∴
 X 2 ( t )   −e  X 2 ( t ) = −e −2t

 India’s No.1 institute for GATE Training  1 Lakh+ Students trained till date  65+ Centers across India
23
EC-GATE-2014 PAPER-02| www.gateforum.com

47. The Bode asymptotic magnitude plot of a minimum phase system is shown in the figure.

26.02

G ( jω )
6.02
( dB)
0
0.1 1 2 10 20
− 6.02
ω ( rad / s ) in log scale

If the system is connected in a unity negative feedback configuration, the steady state error of
the closed loop system, to a unit ramp input, is_________.
Answer: 0.50
Exp:

( −20 dB / dec )
26.02
G ( jw )

dB
6.02
20
0.1 1 2 10 w (r / sec )

−6.02

( −20 db (dec ) )

→ Due to initial slope , it is a type-1 system, and it has non zero velocity error coefficient
(KV )
→ The magnitude plot is giving 0dB at 2r/sec.
Which gives k v
∴kv = 2

A
The steady state error ess =
kv

given unit ramp input; A = 1

1
ess =
2

ess = 0.50

 India’s No.1 institute for GATE Training  1 Lakh+ Students trained till date  65+ Centers across India
24
EC-GATE-2014 PAPER-02| www.gateforum.com

48. Consider the state space system expressed by the signal flow diagram shown in the figure.

C3

1 S−1 S−1 S−1


u x3 x2 x1 c1 y
a3
a2
a1

The corresponding system is


(A) always controllable (B) always observable
(C) always stable (D) always unstable
Answer: (A)
Exp: From the given signal flow graph, the state model is
X  0 1
1 0   X1  0 
      
 X 2  = 0 0 1   X 2  + 0 u
X  a a a1   X 3  1 
 3  3 2
 X1 
Y = [ C1 C 2 C3 ]  X 2 
 X 3 
0 1 0 0
A = 0 0 1  ; B = 0  ;C = [ C1 C 2 C3 ]
 
a 3 a 2 a1  1 
Controllability:
Q c =  B AB A 2 B
0 0 1 
 
QC = 0 1 a1 
1 a1 a 2 + a12 

QC = 1 ≠ 0

Observability
C   C1 C2 C3 
   
Q O =  CA  ⇒  a 3c3 c1 + a 2 c3 c 2 + a1c3 
 
 2
 CA  c 2 a 3 + c3 ( a1a 3 ) a 2 c 2 + c 3 ( a 1a 2 + a 3 ) c1 + a1c 2 + c3 ( a1 + a 2 ) 
2

Q 0 ⇒ depends on a 1 ,a 2 ,a 3 & c1 & c 2 & c 3


.
It is always controllable

 India’s No.1 institute for GATE Training  1 Lakh+ Students trained till date  65+ Centers across India
25
EC-GATE-2014 PAPER-02| www.gateforum.com

49. The input to a 1-bit quantizer is a random variable X with pdf f x ( x ) = 2e −2x for x ≥ 0 and
f x ( x ) = 0 for x < 0 , for x < 0 For outputs to be of equal probability, the quantizer threshold
should be _____.
Answer: 0.35
Exp: one bit
X
Quantizer Q (x )

One bit quantizer will give two levels.


1
Both levels have probability of
2
Pd of input X is

fx ( π )

xT

Let x T be the thsuhold

 x1 x ≥ xT 
Q(x) = 
x2 x < x T 

Where x1 and x 2 are two levels

P {Q ( r ) = x1 } =
1
2

1
⇒ ∫ 2.e −2x dx =
xT 2

e −2x 1
2. =
−2 xT
2
1
−e −2∞ + e −2x T =
2
1
e −2x T =
2
1
− 2x T = ln
2
− 2x T = −0.693
x T = 0.35

 India’s No.1 institute for GATE Training  1 Lakh+ Students trained till date  65+ Centers across India
26
EC-GATE-2014 PAPER-02| www.gateforum.com

50. Coherent orthogonal binary FSK modulation is used to transmit two equiprobable symbol
waveforms s1 ( t ) = α cos 2 πf1t and s2 ( t ) = cos 2 πf 2 t, where α = 4mV. Assume an AWGN
N0
channel with two-sided noise power spectral density = 0.5 × 10−12 W / Hz. Using an
2
1 ∞
optimal receiver and the relation Q ( v ) = ∫ e− u
2
/2
du the bit error probability for a data
2π v

rate of 500 kbps is


( A) Q ( 2) ( B) (
Q 2 2 ) ( C) Q ( 4) ( D) (
Q 4 2 )
Answer: (C)
Exp: For Binary FSK
 E 
Bit error probability = Q 
 N 
 O 

E → Energy per bit [No. of symbols = No. of bits]


A2T 1
E= , A = 4 × 10−3 ,T = [inverse of data rate]
2 500 × 103
16 × 10−6 × 2 × 10−6
⇒E= = 16 × 10−12
2
N 0 = 1 × 10−12
 16 × 10−12 
⇒ Pe = Q   = Q ( 4)
 1 × 10−12 
 

51. The power spectral density of a real stationary random process .X(t) is given by
 1,
Sx ( f ) =  0,w f ≤w
f >w

  1 
The value of the expectation E  π X ( t )  t −   is ___________.
  4w  
Answer: 4
1 
 , f ≤ w
Exp: Given Sx ( f ) =  w 
0 , f ≥ w 
 
w
1
R x ( τ) = ∫ w .e
j2 πft
df
−w

1 e j2 πwt − e − j2 πwt 1  sin ( 2πwt ) 


= =  
w j2πt w πt 
 1 
sin  2πw .  4
  1   1  1  4w =
Now, E  π × ( t ) .x  t −  = π R x   ⇒ π. .
  4w    4w  w 1 1
π.
4w

 India’s No.1 institute for GATE Training  1 Lakh+ Students trained till date  65+ Centers across India
27
EC-GATE-2014 PAPER-02| www.gateforum.com

52. In the figure, M(f) is the Fourier transform of the message signal .m(t) where A = 100 Hz and
B = 40 Hz. Given v(t) = cos ( 2πf c t ) and w ( t ) = cos ( 2 π ( f c + A ) t ) , where f c > A The cutoff
frequencies of both the filters are f C

M (f )
−1

−A −B B A f
m (t) High Pass s(t)
Low Pass
Filter Filter
v (t) w (t)

The bandwidth of the signal at the output of the modulator (in Hz) is _____.
Answer: 60
Exp: m ( t ) ↔ M ( f )
M (f )

f
−A −B B A

After multiplication with V ( t ) = cos ( 2πf c t )

Let w1 ( t ) = m ( t ) .V ( t )
⇒ W1 ( f ) ( specturm of w1 ( t ) ) is

−fc − A − fc − B − fc + B fc + A −fc − A − fc − B − fc + B fc + A
−fc fc

After high pass filter

− fC − A − fC − B fC fC + B fC + A

After multiplication with cos ( 2π ( f c + A ) t ) and low pass filter of cut off f c

 India’s No.1 institute for GATE Training  1 Lakh+ Students trained till date  65+ Centers across India
28
EC-GATE-2014 PAPER-02| www.gateforum.com

f
− (A − B) (A − B)
O
Bandwidth = A − B
= 100 − 40 = 60

53. If the electric field of a plane wave is




) ( )
E ( Z, t ) = x3cos ωt − kz + 30O − y4sin ωt − kz + 45O ( mV / m ) ,
the polarization state of the plane wave is
(A) left elliptical (B) left circular
(C) right elliptical (D) right circular
Answer: (A)
Exp: E ( z1t ) = 3cos ( cot − kz + 3o° ) a x − 4 − sin ( ωt − kz + 45° ) a y
E x = 3cos ( ωt − kz + 30° )
E y = − 4cos ( ωt − kz + 45° )
At z = 0 E x = 3cos ( ωt + 30o )
E y = −4sin ( ωt + 45o )
E x ≠ E y → so Elliptical polarization
Q = 30° −135° = − 105°
∴ left hand elliptical (LEP)

54. In the transmission line shown, the impedance Zin (in ohms) between node A and the ground
is _________.

Z0 = 50Ω, L = 0.5λ
100 Ω 50 Ω
Zin = ?

Answer: 33.33Ω
λ
Exp: Here =
2
(
Zin = λ
2 )=Z L = 50Ω
100
∴ Zin = (100  50 ) = = 33.33Ω
3

 India’s No.1 institute for GATE Training  1 Lakh+ Students trained till date  65+ Centers across India
29
EC-GATE-2014 PAPER-02| www.gateforum.com

55. For a rectangular waveguide of internal dimensions a × b ( a > b ) , the cut-off frequency for
the TE11 mode is the arithmetic mean of the cut-off frequencies for TE10 mode and TE20
mode. If a = 5 cm, the value of b (in cm) is _____.
Answer: 2
2
C 1
Exp: t c10 =  
2 a
1 2
t c10 = K   ; t c20 = K  
a a
1 1
t c11 = K +
a 2 b2
f c10 + f c20
Given t c11 =
2
1 1 K 1 2
K + = +
a 2 b2 2  a a 
1 1 3
2
+ 2 =
a b 2a
1 1 9 1 9 1
+ 2 = ⇒ − + = 2
5 b 4 ( 5) 5 20 b
1
− 0.2 + 0.45 =
b2
1 1
∴ 2
= 2 ⇒ b = 2cm
b 2

 India’s No.1 institute for GATE Training  1 Lakh+ Students trained till date  65+ Centers across India
30
EC-GATE-2014 PAPER-03| www.gateforum.com

Q. No. 1 – 5 Carry One Mark Each

1. “India is a country of rich heritage and cultural diversity.” Which one of the following facts
best supports the claim made in the above sentence?
(A) India is a union of 28 states and 7 union territories.
(B) India has a population of over 1.1 billion.
(C) India is home to 22 official languages and thousands of dialects.
(D) The Indian cricket team draws players from over ten states.
Answer: C
Exp: Diversity is shown in terms of difference language

2. The value of one U.S. dollar is 65 Indian Rupees today, compared to 60 last year. The Indian
Rupee has ____________.
(A) Depressed (B) Depreciated (C) Appreciated (D) Stabilized
Answer: B

3. 'Advice' is ________________.
(A) a verb (B) a noun
(C) an adjective (D) both a verb and a noun
Answer: B

4. The next term in the series 81, 54, 36, 24 … is ________


Answer: 16
2
Exp: 81 − 54 = 27;27 × = 18
3
2
54 − 36 = 18;18 × = 12
3
2
36 − 24 = 12;12 × = 8
3
∴ 24 − 8 = 16

5. In which of the following options will the expression P < M be definitely true?
(A) M < R > P > S (B) M > S < P < F
(C) Q < M < F = P (D) P = A < R < M
Answer: D

Q. No. 6 – 10 Carry Two Marks Each

6. Find the next term in the sequence: 7G, 11K, 13M, ___
(A) 15Q (B) 17Q (C) 15P (D) 17P
Answer: B

 India’s No.1 institute for GATE Training  1 Lakh+ Students trained till date  65+ Centers across India
1
EC-GATE-2014 PAPER-03| www.gateforum.com

7. The multi-level hierarchical pie chart shows the population of animals in a reserve forest. The
correct conclusions from this information are:

Beetle Tiger
Re d − ant

Elephant
Mammal
Honey
− bee In sec t
Leopard
Re ptile
Snake
Moth Bird
Crocadile
Hawk Drongo
Bulbul
Butterfly

(i) Butterflies are birds


(ii) There are more tigers in this forest than red ants
(iii) All reptiles in this forest are either snakes or crocodiles
(iv) Elephants are the largest mammals in this forest
(A) (i) and (ii) only (B) (i), (ii), (iii) and (iv)
(C) (i), (iii) and (iv) only (D) (i), (ii) and (iii) only
Answer: D
Exp: It is not mentioned that elephant is the largest animal

8. A man can row at 8 km per hour in still water. If it takes him thrice as long to row upstream,
as to row downstream, then find the stream velocity in km per hour.
Answer: 4
Exp: 4 km/hr.
Speed of man=8
Left distance =d
d
Time taken=
8
Upstream:
Speed of stream=s
⇒ speed upstream = S' = (8 − s)
 d 
t' =  
8−s 
Downstream:
d
Given speed downstream = t '' =
8+s

 India’s No.1 institute for GATE Training  1 Lakh+ Students trained till date  65+ Centers across India
2
EC-GATE-2014 PAPER-03| www.gateforum.com

⇒ 3t ' = t ''
3d d
⇒ =
8−s 8+s
3d d
⇒ =
8−s 8+s
⇒ s = 4km / hr

9. A firm producing air purifiers sold 200 units in 2012. The following pie chart presents the
share of raw material, labour, energy, plant & machinery, and transportation costs in the total
manufacturing cost of the firm in 2012. The expenditure on labour in 2012 is Rs. 4,50,000. In
2013, the raw material expenses increased by 30% and all other expenses increased by 20%.
If the company registered a profit of Rs. 10 lakhs in 2012, at what price (in Rs.) was each air
purifier sold?

Trans
Labour
portat
ion 15%

10%
Plant and Raw Material
machinery 20%
30%
Energy
25%

Answer: 20,000
15
Exp: Total expenditure= = x = 4,50,000
100
x=3×106
Profit=10 lakhs
So, total selling price =40,00,000 … (1)
Total purifies=200 … (2)
S.P of each purifier=(1)/(2)=20,000

10. A batch of one hundred bulbs is inspected by testing four randomly chosen bulbs. The batch
is rejected if even one of the bulbs is defective. A batch typically has five defective bulbs.
The probability that the current batch is accepted is _________
Answer: 0.8145
95
Exp: Probability for one bulb to be non defective is
100
4
 95 
∴ Probabilities that none of the bulbs is defectives   = 0.8145
 100 

 India’s No.1 institute for GATE Training  1 Lakh+ Students trained till date  65+ Centers across India
3
EC-GATE-2014 PAPER-03| www.gateforum.com

Q.No. 1 – 25 Carry One Mark Each

1. The maximum value of the function f(x) = ln(1 + x)- x (where .x > - 1) occurs at x=______.
Answer: 0
1
Exp: f1 (x) = 0 ⇒ −1 = 0
1+ x
−x
⇒ =0⇒x =0
1+ x
−1
and f 11 ( x ) = < 0 at x = 0
(1 + x )
2

2. Which ONE of the following is a linear non-homogeneous differential equation, where x and
y are the independent and dependent variables respectively?
dy dy
( A) + xy = e − x ( B) + xy = 0
dx dx
dy dy − y
( C) + xy = e − y ( D) + e = e− y = 0
dx dx
Answer: A
dy
Exp: (A) + xy = e − x is a first order linear equation (non-homogeneous)
dx
dy
(B) + xy = 0 is a first order linear equation (homogeneous
dx
(C), (D) are non linear equations

3. Match the application to appropriate numerical method.


Application Numerical |Method
P1: Numerical integration M1: Newton-Raphson Method
P2: Solution to a transcendental equation M2: Runge-Kutta Method
P3: Solution to a system of linear equations M3: Simpson’s 1/3-rule
P4: Solution to a differential equation M4: Gauss Elimination Method
(A) P1—M3, P2—M2, P3—M4, P4—M1 (B) P1—M3, P2—M1, P3—M4, P4—M2
(C) P1—M4, P2—M1, P3—M3, P4—M2 (D) P1—M2, P2—M1, P3—M3, P4—M4
Answer: B
Exp: P1 − M3,P2 − M1, P3 − M4,P4 − M2

4. An unbiased coin is tossed an infinite number of times. The probability that the fourth head
appears at the tenth toss is
(A) 0.067 (B) 0.073 (C) 0.082 (D) 0.091

 India’s No.1 institute for GATE Training  1 Lakh+ Students trained till date  65+ Centers across India
4
EC-GATE-2014 PAPER-03| www.gateforum.com

Answer: C
Exp: P[fourth head appears at the tenth toss] = P [getting 3 heads in the first 9 tosses and one
head at tenth toss]
  1 9   1  21
=  9 C3 .    ×   = = 0.082
  2    2  256

5. If z = xyln(xy), then
∂z ∂z ∂z ∂z
( A) x +y =0 ( B) y =x
∂x ∂y ∂x ∂y
∂z ∂z ∂z ∂z
( C) x =y ( D) y +x =0
∂x ∂y ∂x ∂y
Answer: C
∂z  1 
Exp: = y  x × × y + ln xy  = y (1 + ln xy )
∂x  xy 
∂z ∂z ∂z
and = x (1 + ln xy ) ⇒ x =y
∂y ∂x ∂y

6. A series RC circuit is connected to a DC voltage source at time t = 0. The relation between


the
source voltage VS, the resistance R, the capacitance C, and the current i(t) is given below:
1 t
Ve = Ri ( t ) + i ( u ) du
c ∫0
Which one of the following represents the current f(t)?

( A) ( B)

i (t) i (t)

0 t
0 t

( C) ( D)
i (t) i (t)

0 t 0 t

 India’s No.1 institute for GATE Training  1 Lakh+ Students trained till date  65+ Centers across India
5
EC-GATE-2014 PAPER-03| www.gateforum.com

Answer: A
Exp: In a series RC circuit,
VS
→ Initially at t = 0, capacitor charges with a current of and in steady state at t = ∞ ,
R
capacitor behaves like open circuit and no current flows through the circuit
→ So the current i(t) represents an exponential decay function

i (t )

0 →t

7. In the figure shown, the value of the current I (in Amperes) is __________.
5Ω 5Ω
I

5V ± ↑ 1A 10 Ω

Answer: 0.5
Exp:
V
I
5Ω 5Ω
+
5V − 1A 10Ω

V−5 V
Apply KCL at node V, −1 + = 0
5 15
30
⇒ V = volts
4
V 2
⇒ current I = ⇒ ⇒ 0.50 Amperes
15 4

8. In MOSFET fabrication, the channel length is defined during the process of


(A) Isolation oxide growth
(B) Channel stop implantation
(C) Poly-silicon gate patterning
(D) Lithography step leading to the contact pads
Answer: C

 India’s No.1 institute for GATE Training  1 Lakh+ Students trained till date  65+ Centers across India
6
EC-GATE-2014 PAPER-03| www.gateforum.com

9. A thin P-type silicon sample is uniformly illuminated with light which generates excess
carriers. The recombination rate is directly proportional to
(A) The minority carrier mobility
(B) The minority carrier recombination lifetime
(C) The majority carrier concentration
(D) The excess minority carrier concentration
Answer: D
Exp: (
Recombination rate, R = B n n o + n n' )( P no + Pn' )
n n0 & Pn0 = Electron and hole concentrations respectively under thermal equilibrium
n 'n & p 'n = Excess elements and hole concentrations respectively

kT
10. At T = 300 K, the hole mobility of a semiconductor µ P = 500cm 2 / V − s and = 26 mV.
q
The hole diffusion constant D P in cm2/s is ________
Answer: 13
Exp: From Einstein relation,
D P kJ
=
µp q
⇒ D P = 26 mV × 500cm 2 / v − s = 13cm 2 / s

11. The desirable characteristics of a transconductance amplifier are


(A) High input resistance and high output resistance
(B) High input resistance and low output resistance
(C) Low input resistance and high output resistance
(D) Low input resistance and low output resistance
Answer: A
Exp: Transconductance amplifier must have z i = ∞ and z 0 = ∞ ideally

12. In the circuit shown, the PNP transistor has VBE = 0.7 and β = 50. Assume that R B = 100kΩ
For V0 to be 5 V, the value of R C ( in kΩ ) _________________

RC

V0

RB VEE = 10 V

 India’s No.1 institute for GATE Training  1 Lakh+ Students trained till date  65+ Centers across India
7
EC-GATE-2014 PAPER-03| www.gateforum.com

Answer: 1.075
Exp: KVL in base loop gives,
10 − 0.7
IB = = 93 µA
100K
⇒ IC = β IB = 50 × 93 µA = 4.65mA

from figure, V0 = IC R C

V0 5V
⇒ RC = = = 1.075 Ω
IC 4.65 mA

13. The figure shows a half-wave rectifier. The diode D is ideal. The average steady-state current
(in Amperes) through the diode is approximately ____________.

D
C
10sin ωt ~ R
4 mF
f = 50 Hz 100 Ω

Answer: 0.09
Idc
Exp: Vdc = Vm −
4fc
Idc
Idc R L = Vm −
4fc
 1 
Idc  R L + = Vm
 4fc 
10
⇒ Idc = = 0.09A
1
100 +
4 × 50 × 4 × 10−3

14. An analog voltage in the range 0 to 8 V is divided in 16 equal intervals for conversion to 4-bit
digital output. The maximum quantization error (in V) is _________________
Answer: 0.25
step − size
Exp: Maximum quantization error is
2
8−0 1
step − size = = = 0.5V
16 2
Quantization error = 0.25 V

 India’s No.1 institute for GATE Training  1 Lakh+ Students trained till date  65+ Centers across India
8
EC-GATE-2014 PAPER-03| www.gateforum.com

15. The circuit shown in the figure is a

D Q Q
D Latch D Latch
En Q En Q

Clk

(A) Toggle Flip Flop (B) JK Flip Flop


(C) SR Latch (D) Master-Slave D Flip Flop
Answer: D
Exp: Latches are used to construct Flip-Flop. Latches are level triggered, so if you use two latches
in cascaded with inverted clock, then one latch will behave as master and another latch which
is having inverted clock will be used as a slave and combined it will behave as a flip-flop. So
given circuit is implementing Master-Slave D flip-flop

16. Consider the multiplexer based logic circuit shown in the figure.

W 0
MUX 0
1 MUX F
1
S1
S2

Which one of the following Boolean functions is realized by the circuit?


( A) F = WS1 S2 ( B) F = WS1 + WS2 + S1S2

( C) F = W + S1 + S2 ( D) F = W ⊕ S1 ⊕ S2
Answer: D
Exp:
W
0
MUX
Y
1 0
MUX F
1

S1
S2

Output of first MUX = ws1 + ws1 = w ⊕ s1


Let Y = w ⊕ s1
Output of second MUX = Ys2 + Y s 2
= Y ⊕ s2
= w ⊕ s1 + s 2

 India’s No.1 institute for GATE Training  1 Lakh+ Students trained till date  65+ Centers across India
9
EC-GATE-2014 PAPER-03| www.gateforum.com

17. Let x(t)= cos (10πt ) + cos ( 30πt ) be sampled at 20 Hz and reconstructed using an ideal low-
pass filter with cut-off frequency of 20 Hz. The frequency/frequencies present in the
reconstructed signal is/are
(A) 5 Hz and 15 Hz only (B) 10 Hz and 15 Hz only
(C) 5 Hz, 10 Hz and 15 Hz only (D) 5 Hz only
Answer: (A)
Explanation: x ( t ) = cos (10πt ) + cos ( 30πt ) , Fs = 20Hz
Spectrum of x(t)

−15 − 5 5 15 t

Spectrum of sampled version of x(t)

−35 25 − 15 − 5 5 15 25 35

After LPF, signal will contain 5 and 15Hz component only

( z − b) , where H e = 1,
−1

18. For an all-pass system H (z) =


(1 − az ) −1 ( ) − jω
for all ω .If

Re ( a ) ≠ 0, Im ( a ) ≠ 0, then b equals
(A) a (B) a* (C) 1/a* (D) 1/a
Answer: (B)
1 1
Exp: For an all pass system, pole = or zero =
zero * pole*
pole = a
1
zero =
b
1 1
⇒ = or b = a *
b a*

19. A modulated signal is y(t) = m.(t)cos(40000 πt ), where the baseband signal m(t) has
frequency components less than 5 kHz only. The minimum required rate (in kHz) at which
y(t) should be sampled to recover m(t) is __________________.

 India’s No.1 institute for GATE Training  1 Lakh+ Students trained till date  65+ Centers across India
10
EC-GATE-2014 PAPER-03| www.gateforum.com

Answer: 10 KHz.
Exp: Since m(t) is a base band signal with maximum frequency 5 KHz, assumed spreads as
follows:

M(f )

f
m(t) M(f )

−5k +5k f (Hz)

*1
∵ y(t) = m(t) cos(40000π t) 
7
→ m(f ) [δ(f − 20k) + δ(f + 20k)]
2
1
∴ y(f ) = [ M(f − 20k) + M(f + 20 k)]
2
Thus the spectrum of the modulated signal is as follows:

y(f )

15k 25k f (Hz)


−25k −20k −15k 20k

If y(t) is sampled with a sampling frequency ‘fs’ then the resultant signal is a periodic
extension of successive replica of y(f) with a period ‘fs’.
It is observed that 10 KHz and 20 KHz are the two sampling frequencies which causes a
replica of M(f) which can be filtered out by a LPF.
Thus the minimum sampling frequency (fs) which extracts m(t) from g(f) is 10 KHz.

20. Consider the following block diagram in the figure.

R (s) + + C ( s)
G1 G2
+ +

C ( s)
The transfer function is
R ( s)
G1G 2 G1
( A) ( B) G1G 2 + G1 + 1 ( C) G1G 2 + G 2 + 1 ( D)
1 + G1G 2 1 + G1G 2

 India’s No.1 institute for GATE Training  1 Lakh+ Students trained till date  65+ Centers across India
11
EC-GATE-2014 PAPER-03| www.gateforum.com

Answer: C
Exp: By drawing the signal flow graph for the given block diagram
G1 G2 1
C (s)
R (s) 1
1
1
Number of parallel paths are three
Gains P1 = G1G 2 , P2 = G 2 , P3 = 1
By mason’s gain formula,
C(s)
= P1 + P2 + P3
R (s)
⇒ G1G 2 + G 2 + 1

21. The input −3e2 t u ( t ) , where u(t) is the unit step function, is applied to a system with transfer
s−2
function . . If the initial value of the output is -2, then the value of the output at steady
s+3
state is__________________.
Answer: 0
Exp: 1
Y (s ) S− 2
=
X (s ) S+3
⇒ SY ( s ) + 3Y ( s ) = S × ( s ) − 2X ( s )
Due to initial condition, we can write above equation as
Sy ( s ) − y ( 0 ) + 3y ( s ) = sx ( s ) − x ( 0 − ) − 2x ( s )

y ( 0− ) = −2, x ( 0− ) = 0  x ( t ) = 3e2t u ( t ) 
 −3 
⇒ Sy ( s ) + 2 + 3y ( s ) = ( s − 2 )  
s−2
−5
( s + 3) y ( s ) = −3 − 2 ⇒ y ( s ) =
5+3
⇒ y ( t ) = −5e u ( t )
−3t

y ( ∞ )( steady sate ) = 0
Exp: 2
s−2
H (s) = ;X ( t ) = −3e2 t .u ( t )
s+3
−3 −3
∴ X (s) = ⇒ Y (s) =
s−2 s+3
−3s
y ( t ) at t =∞ ⇒ y ( ∞ ) = lim S.y ( s ) = lim
s →0 s →0 s+3
y(∞) = 0

 India’s No.1 institute for GATE Training  1 Lakh+ Students trained till date  65+ Centers across India
12
EC-GATE-2014 PAPER-03| www.gateforum.com

22. The phase response of a passband waveform at the receiver is given by


φ ( f ) = −2πα ( f − f c ) − 2πβ f c
Where fc is the centre frequency, and α and β are positive constants. The actual signal
propagation delay from the transmitter to receiver is
α −β αβ
(A) ( B) ( C) α ( D) β
α+β α +β
Answer: C
Exp: Phase response of pass band waveform
φ ( f ) = −2πα ( f − f c ) − 2πβ f c
− dφ ( f )
Group delay t y = =α
2π df
Thus ' α ' is actual signal propagation delay from transmitter to receiver

23. Consider an FM signal f ( t ) = cos [ 2 πf c t + β1 sin 2 πf1t + β 2 sin 2πf 2 t.] . The maximum
deviation of the instantaneous frequency from the carrier frequency fc is
( A) β1f1 + β2f 2 ( B) β1f 2 + β 2f1 ( C) β1 + β2 ( D) f1 + f 2
Answer: A
Exp: Instantaneous phase φi ( t ) = 2π f c t + β1 sin 2πf1 + β 2 sin 2πf 2 t
d 1
Instantaneous frequency f i ( t ) = φi ( t ) ×
dt 2π
= f c + β1f1 cos 2πf1t + β2 f 2 cos 2πf 2 t
Instantaneous frequency deviation = β1f1 cos 2 πf1 t + β2 f 2 cos 2 πf 2 t
Maximum ∆f = β1f1 + β2 f 2

24. Consider an air filled rectangular waveguide with a cross-section of 5 cm × 3 cm. For this
waveguide, the cut-off frequency (in MHz) of TE21 mode is _________.
Answer: 7810MHz.
2 2
C 2 1
Exp: f c ( TE 21 ) =   + 
2 9 b

3 × 1010  2   1 
2 2

=   + 
2  5 3
= 1.5 × 1010 0.16 + 0.111
= 0.52 × 1.5 × 1010
= 7.81 GHz
= 7810 MHz.

 India’s No.1 institute for GATE Training  1 Lakh+ Students trained till date  65+ Centers across India
13
EC-GATE-2014 PAPER-03| www.gateforum.com

25. In the following figure, the transmitter Tx sends a wideband modulated RF signal via a
coaxial cable to the receiver Rx. The output impedance ZT of Tx, the characteristic
impedance Z0 of the cable and the input impedance ZR of Rx are all real.

Transmitter Re ceiver
Characteristic Im pedance = Z0

ZT ZR
TX RX

Which one of the following statements is TRUE about the distortion of the received signal
due to impedance mismatch?
(A) The signal gets distorted if ZR ≠ Z0, irrespective of the value of ZT
(B) The signal gets distorted if ZT ≠ Z0, irrespective of the value of ZR
(C) Signal distortion implies impedance mismatch at both ends: ZT ≠ Z0 and ZR ≠ Z0
(D) Impedance mismatches do NOT result in signal distortion but reduce power transfer
efficiency
Answer: C
Exp: Signal distortion implies impedance mismatch at both ends. i.e.,
ZT ≠ Z 0
ZR ≠ Z0

Q. No. 26 – 55 Carry Two Marks Each

26. The maximum value of f(x)=2x3 – 9x2 +12x - 3 in the interval 0 ≤ x ≤ 3 is _______.
Answer: 6
Exp: f 1 ( x ) = 6x 2 − 18x + 12 = 0 ⇒ x = 1, 2 ∈ [ 0,3]

Now f ( 0 ) = −3 ; f ( 3) = 6 and f (1) = 2 ; f ( 2 ) = 1

Hence, f(x) is maximum at x = 3 and the maximum value is 6

27. Which one of the following statements is NOT true for a square matrix?
(A) If A is upper triangular, the eigenvalues of A are the diagonal elements of it
(B) If A is real symmetric, the eigenvalues of A are always real and positive
(C) If A is real, the eigenvalues of A and AT are always the same
(D) If all the principal minors of A are positive, all the eigenvalues of A are also positive
Answer: B

 India’s No.1 institute for GATE Training  1 Lakh+ Students trained till date  65+ Centers across India
14
EC-GATE-2014 PAPER-03| www.gateforum.com

 −1 1
Exp: Consider, A  which is real symmetric matrix
1 − 1

Characteristic equation is A − λI = 0 ⇒ (1 + λ ) − 1 = 0
2

⇒ λ + 1 = ±1
∴λ = 0, − 2 ( not positive )

(B) is not true


(A), (C), (D) are true using properties of eigen values

28. A fair coin is tossed repeatedly till both head and tail appear at least once. The average
number of tosses required is __________________.
Exp: Let the first toss be Head.
Let x denotes the number of tosses( after getting first head) to get first tail.
We can summarize the even as:
Event x Probability(p(x))
(After getting first H)
T 1 1/2
HT 2 1/2*1/2=1/4
HHT 3 1/8
and so on………..

1 1 1
E ( x ) = ∑ xp ( x ) = 1x + 2x + 3x 
x =1 2 4 8
1 1 1
Let, S =1x + 2x + 3x  ( I)
2 4 8
1 1 1 1
⇒ S= + 2x + 3x  ( II )
2 4 8 16
( I − II ) gives
 1 1 1 1 1
1 −  S = + + + + 
 2 2 4 8 16
1
1
⇒ S = 2 =1
2 1
1−
2
⇒S= 2
⇒ E(x) = 2

i.e. The expected number of tosses (after first head) to get first tail is 2 and same can be
applicable if first toss results in tail.

Hence the average number of tosses is 1+2 = 3.

 India’s No.1 institute for GATE Training  1 Lakh+ Students trained till date  65+ Centers across India
15
EC-GATE-2014 PAPER-03| www.gateforum.com

29. Let X1, X2, and X3 be independent and identically distributed random variables with the
uniform distribution on [0, 1]. The probability P{X1+ X2 ≤ X3} is ________.
Answer: 0.16
Exp: Given x1 x 2 and x 3 be independent and identically distributed with uniform distribution on
[0,1]
Let z = x1 + x 2 − x 3
⇒ P {x1 + x 2 ≤ x 3 } = P {x1 + x 2 − x 3 ≤ 0}
= P {z ≤ 0}
Let us find probability density function of random variable z.
Since Z is summation of three random variable x1 , x 2 and − x 3
Overall pdf of z is convolution of the pdf of x1 x 2 and − x 3
pdf of {x1 + x 2 } is

O 1 2

1
pdf of − x 3 is

−1

3 0
( z + 1) ( z + 1)
0 2
1
P {z ≤ 0} = ∫ dz = = = 0.16
−1
2 6 6
−1

30. Consider the building block called ‘Network N’ shown in the figure.
Let C = 100µF and R = 10kΩ
Network N

+ +
C
V1 ( s) R V2 ( s )

− −

Two such blocks are connected in cascade, as shown in the figure.

+ + +
V1 ( s) Network N Network N V3 ( s )
− − −

 India’s No.1 institute for GATE Training  1 Lakh+ Students trained till date  65+ Centers across India
16
EC-GATE-2014 PAPER-03| www.gateforum.com

v 3 ( s)
The transfer function of the cascaded network is
v1 ( s )
2
s s2  s  s
( A) ( B) ( C)   ( D)
1+ s 1 + 3s + s2 1+ s 2+s
Answer: B
Exp: Two blocks are connected in cascade, Represent in s-domain,

+ 1 1 +
SC SC
V1 ( s ) R R V3 ( s )

− −

V3 ( s ) R . R
=
V1 ( s ) 1 1   1 
R+R+ +R + R
sc  SC   SC 
R. R
=
1 1 R
.
SC SC
 2R ( SC ) + 1 +
SC
[1 + RSC]
S2 C 2 .R.R
=
1 + 2R ( SC )  + RSC + R S C
2 2 2

S2 .100 × 100 × 10−6 × 10−6 × 10 × 10 × 103 × 103


=
S2 × 100 × 106 × 104 × 10−12 + 3S + 100 × 10−6 × 104 + 1
V3 ( s ) S2
=
V1 ( s ) 1 + 3S + S2

31. In the circuit shown in the figure, the value of node voltage V2 is

10∠0O V
+ −
V1 V2
4Ω
4∠0OA 6Ω j6Ω
− j3Ω

(A) 22 + j 2 V (B) 2 + j 22 V (C) 22 – j 2 V (D) 2 – j 22 V

 India’s No.1 institute for GATE Training  1 Lakh+ Students trained till date  65+ Centers across India
17
EC-GATE-2014 PAPER-03| www.gateforum.com

Answer: D
4Ω
Exp: Super node
0
V 10 0 V2
V
+ −

4 0o A 6Ω j6 Ω
− j3Ω

KVL for V1 & V2 :


V1
V2
+ −
+ +
− 10 0 o −

V1 − V2 = 10 0o ....(1)
V1 = V2 + 10 0o

KCL at super node:


V1 V2 V2
−4 0o + + + =0 ... ( 2 )
− j3 6 j6
V1 V2 V2
+ + = 4 0o
− j3 V6 j6
V2 + 10 0o V2 V2
from (1) & ( 2 ) , + + = 4 0o
− j3 6 j6
 1 1 1 10
V2  + +  = 4 0o +
 − j3 6 j6  j3
∴ V2 = ( 2 − j22 ) Volts

32. In the circuit shown in the figure, the angular frequency ω (in rad/s), at which the Norton
equivalent impedance as seen from terminals b-b' is purely resistive, is
__________________.
1Ω 1F
b
+
10cos ωt ~
( Volts) − 0.5 H

b'

Answer: 2 r/sec

 India’s No.1 institute for GATE Training  1 Lakh+ Students trained till date  65+ Centers across India
18
EC-GATE-2014 PAPER-03| www.gateforum.com

Exp: Norton’s equivalent impedance


1
1* jω. b
ZN = 2 + 1 1Ω 1F
1 jω.1
1 + jω. 0.5H
2
jω 1
= + b1
Zu
2 + jω jω
( 2 − ω ) + jω ⇒ Z
2 ( ω2 − 2 ) − jω .  ω2 + 2 jω
ZN = =    
 2 jω − ω   ω + 4ω 
2 N 4 2

Equating imaginary term to zero i.e., ω3 − 4ω = 0


⇒ ω ( ω2 − 4 ) = 0 ⇒ ω = 2 r / sec

33. For the Y-network shown in the figure, the value of R 1 ( in Ω ) in the equivalent ∆ -network is
_____________________.
R1

5Ω 3Ω

7.5Ω

Answer: 10Ω R1
Exp:
5Ω 3Ω

7.5Ω

R1 =
( 7.5)( 5) + ( 3)( 5 ) + ( 7.5)( 3) Ω
7.5
R 1 = 10Ω

34. The donor and accepter impurities in an abrupt junction silicon diode are 1 x 1016 cm-3 and 5
x 1018 cm-3, respectively. Assume that the intrinsic carrier concentration in silicon ni = 1.5 x
kT
1010 cm-3 at 300 K, = 26 mV and the permittivity of silicon εsi = 1.04 × 10−12 F / cm. The
q
built-in potential and the depletion width of the diode under thermal equilibrium conditions,
respectively, are
(A) 0.7 V and 1 x 10-4 cm (B) 0.86 V and 1 x 10-4 cm
(C) 0.7 V and 3.3 x 10-5 cm (D) 0.86 V and 3.3 x 10-5 cm

 India’s No.1 institute for GATE Training  1 Lakh+ Students trained till date  65+ Centers across India
19
EC-GATE-2014 PAPER-03| www.gateforum.com

Answer: D
 
NA ND  5 × 1018 × 1 × 1016 
Exp: Vbi = VT ln = 26 mv ln
ni2  (1.5 × 1010 )2 
 
= 0.859V
2εS Vbi  N A + N D  −5
W=   = 3.34 × 10 cm
q  NA ND 

35. The slope of the ID vs VGS curve of an n-channel MOSFET in linear regime is 10−3 Ω −1 at
VDS = 0.1V. . For the same device, neglecting channel length modulation, the slope of the
( )
I D vs VGS curve in A / V under saturation regime is approximately ___________.

Answer: 0.07
 V 2
Exp: In linear region, I D = k ( VGS − VT ) VDS − DS 
 2 

∂I D VDS2
= 10−3 = kVDS ∵ VDS is small, is neglected
∂VGS 2
10−3
⇒K= = 0.01
0.1
1
In saturation region, I D = k ( VGS − VT )
2

2
k
ID = ( VGS − VT )
2

∂ ID k 0.01
= = = 0.07
∂VGS 2 2

36. An ideal MOS capacitor has boron doping-concentration of 1015 cm-3 in the substrate. When a
gate voltage is applied, a depletion region of width 0.5 µm is formed with a surface (channel)
potential of 0.2 V. Given that εo = 8.854 × 10-14 F/cm and the relative permittivities of silicon
and silicon dioxide are 12 and 4, respectively, the peak electric field (in V/µm) in the oxide
region is __________________.
Answer: 2.4
2 × 0.2
Exp: Es = = 0.8 v / µm
0.5
Es
E ox = E s = 2.4 v / µm
E ox

 India’s No.1 institute for GATE Training  1 Lakh+ Students trained till date  65+ Centers across India
20
EC-GATE-2014 PAPER-03| www.gateforum.com

37. In the circuit shown, the silicon BJT has β = 50 . Assume VBE =0.7 V and VCE(sat) = 0.2 V.
Which one of the following statements is correct?
10 V

RC

50kΩ
5V
RB

(A) For RC = 1 kΩ, the BJT operates in the saturation region


(B) For RC = 3 kΩ, , the BJT operates in the saturation region
(C) For RC =20 kΩ, , the BJT operates in the cut-off region
(D) For RC =20 kΩ, , the BJT operates in the linear region
Answer: B
Exp: KVL in base loop,
5 − I B ( 50 k ) − 0.7 = 0
5 − 0.7
IB = = 80 µA
50 k
⇒ IC = βIB = 50 × 86 µA = 4.3mA
10 − VCE ( sat ) 10 − 0.2
∴RC = =
IC 4.3mA
R C = 2279 Ω and the BJT is in saturation

38. Assuming that the Op-amp in the circuit shown is ideal, VO is given by

3R
V1 −
R VO
V2 +
2R R

5 5 3 7 11
( A) V1 − 3V2 ( B) ZV1 − V2 ( C) − V1 + V2 ( D) − 3V1 + V2
2 2 2 2 2
Answer: D
Exp: Virtual ground and KCL at inverting terminal gives
3R
V2 − V1 V2 V2 − V0
+ + =0
R 2R 3R R V2
• • −
V1 • Vo
V0 V2 V2 V2 V1 • +
= + + − 2R V2 R
3R R 3R 2R R
11
V0 = −3V1 + V2
2

 India’s No.1 institute for GATE Training  1 Lakh+ Students trained till date  65+ Centers across India
21
EC-GATE-2014 PAPER-03| www.gateforum.com

39. For the MOSFET M1 shown in the figure, assume W/L = 2, VDD = 2.0 V, µ n Cox = 100µA / V 2
and VTH = 0.5 V. The transistor M1 switches from saturation region to linear region when Vin
(in Volts) is_________________.

VDD

R = 10 kΩ

Vout

Vin M1

Answer: 1.5
Exp: Transistor m1 switch from saturation to linear
⇒ VDS = VGS − VT ; where VDS = V0 and VGS = Vi

∴ VDS = V0 = Vi − VT

1 w
Drain current ID = µ n cos ( VGS − VT )
2

2 L
VDD − Vo 1
= × 100 × 10−6 × 2 ( VGS − 0.5 )
2

10K 2
2 − ( Vi − 0.5 )
= 100 × 10−6 ( Vi − 0.5 )
2

10 K

⇒ Vi = 1.5V

40. If WL is the Word Line and BL the Bit Line, an SRAM cell is shown in

WL
WL
VDD
VDD BL
BL BL
BL

(A) ( B)

 India’s No.1 institute for GATE Training  1 Lakh+ Students trained till date  65+ Centers across India
22
EC-GATE-2014 PAPER-03| www.gateforum.com

WL WL
VDD VDD
BL BL BL
BL

( C) ( D)

Answer: B
Exp: For an SRAM construction four MOSFETs are required (2-PMOS and 2-NMOS) with
interchanged outputs connected to each CMOS inverter. So option (B) is correct.

41. In the circuit shown, W and Y are MSBs of the control inputs. The output F is given by

4 :1 MUX 4 :1 MUX

I0 I0

I1 I1 F
VCC Q
Q I2
I2

I3 I3

W X Y Z

(A) F = W X + WX + Y Z ( B) F = W X + WX + YZ

( C) F = W XY + WXY ( D) (
F = W + X YZ )
Answer: C
Exp: I0 I0

I1 4 :1 I1 4 :1
Vcc MUX Q MUX Q F
I2 I2

I3 I3

W X Y Z

The output of the first MUX = W × Vcc + WX.Vcc


WX + WX (∵ Vcc = log ic1)
=W ⊕ X
Let Q = W ⊕ X

 India’s No.1 institute for GATE Training  1 Lakh+ Students trained till date  65+ Centers across India
23
EC-GATE-2014 PAPER-03| www.gateforum.com

The output of the second MUX = Q.Y Z + Q. Y Z


= Q.Y ( Z + Z )
= Q.Y.1 = Q.Y
Put the value of Q in above expression
= ( WX + WX ) .Y
= W X.Y + WX.Y

42. If X and Y are inputs and the Difference (D = X – Y) and the Borrow (B) are the outputs,
which
one of the following diagrams implements a half-subtractor?

Y I0 X I0
2 :1 2 :1
MUX D MUX D
I1
( A) I1
( B)
S S
X Y
S S
Y I0 X I0
2 :1 2 :1
MUX B MUX B
I1 I1

Y I0 X I0
2 :1 2 :1
MUX B MUX B
( C) I1
( B)
I1
S S
X Y
S S
Y I0 X I0
2 :1 2 :1
MUX D MUX D
I1 I1

Answer: A
Exp:
X Y D B
0 0 0 0
0 1 1 1
1 0 1 0
1 1 0 0

So, D = X ⊕ Y = XY + XY and B = X.Y

 India’s No.1 institute for GATE Training  1 Lakh+ Students trained till date  65+ Centers across India
24
EC-GATE-2014 PAPER-03| www.gateforum.com

Y I0
2 :1
D = X. Y + XY
MUX
=X ⊕ Y
I1

Y I0
2 :1
B = X. Y + X.0
MUX
= X.Y + 0
I1
= X .Y

Let H1 ( z ) = (1 − pz −1 ) , H 2 ( z ) = (1 − qz −1 ) , H ( z ) = H1 ( z ) + r H 2 ( z ) . The quantities p, q, r


−1 −1
43.
1 1
are real numbers. Consider p = ,q = , r < 1. If the zero of H(z) lies on the unit circle, then
2 4
r = ________
Answer: -0.5

H1 ( z ) = (1 − Pz −1 )
−1
Exp:

H 2 ( z ) = (1 − qz −1 )
−1

1 1 1 − qz −1 + r (1 − Pz −1 ) (1 + r ) − ( q + rp ) z −1
H (z) = +r = =
1 − Pz −1 (1 − qz −1 ) (1 − Pz −1 )(1 − Pz −1 ) (1 − Pz −1 )(1 − Pz −1 )
q + rp
zero of H ( z ) =
1+ r
Since zero is existing on unit circle
q + rp q + rp
⇒ = 1 or = −1
1+ r 1+ r
1 r 1 r
− + − +
4 2 = 1 or 4 2 = −1
1+ r 1+ r
1 r 1 r
− + =1+ r or − + = −1 − r
4 2 4 2
5 r 5 3 −3r
⇒r=− ⇒ =− or = r = − 1 ⇒ r = −0.5
2 2 4 4 2 2

5
r=− is not possible
2

 India’s No.1 institute for GATE Training  1 Lakh+ Students trained till date  65+ Centers across India
25
EC-GATE-2014 PAPER-03| www.gateforum.com

1
44. Let h(t) denote the impulse response of a causal system with transfer function . Consider
s +1
the following three statements.
S1: The system is stable.
h ( t + 1)
S2: is independent of t for t 0.
h ( t)
S3: A non-causal system with the same transfer function is stable.
For the above system,
(A) Only S1 and S2 are true (B) only S2 and S3 are true
(C) Only S1 and S3 are true (D) S1, S2 and S3 are true
Answer: A
1
Exp: h ( t ) ↔ H (s) = ⇒ h ( t ) = e− t u ( t )
s +1
S1 : System is stable (TRUE)
Because h(t) absolutely integrable
h ( t + 1)
S2 : is independent of time (TRUE)
h(t)
− ( t +1)
e
⇒ e −1 (independent of time)
e− t
S3 : A non-causal system with same transfer function is stable
1
↔ −e − t u ( − t ) (a non-causal system) but this is not absolutely integrable thus
s +1
unstable.
Only S1 and S2 are TRUE

1
45. The z-transform of the sequence x[n] is given by X ( z ) = , , with the region of
(1 − 2z )
−1 2

convergence z > 2. Then, x [ 2 ] is ______.


Answer: 12
Exp(1):
1 1 1
X ( z) = =
(1 − 2z ) −1 2
(1 − 2z ) (1 − 2z−1 )
−1

x [ n ] = 2n u [ n ] * 2n u [ n ]
n
x [ n ] = ∑ 2K.2(
n −k )

k =0
2
⇒ x [ 2] = ∑ 2k.2(
2− k )
= 20.22 + 21.21 + 22.20 = 4 + 4 + 4 = 12
k =0

 India’s No.1 institute for GATE Training  1 Lakh+ Students trained till date  65+ Centers across India
26
EC-GATE-2014 PAPER-03| www.gateforum.com

Exp(2):
1 Z2
X (z) = =
(1 − 2Z ) ( Z − 2)
−1 2 2

 
 
−1  Z Z 
X(n) = Z .
Z−2  Z − 2
 ↓ ↓ 
 u ( z ) 
v( z ) 

= ∑ u m .Vn − m ( u sin g conduction theorem and u n = 2n ; v n = 2n )


n

m =0

n
= ∑ 2m.2n − m = 2n ( n + 1)
m =0

∴ x ( 2 ) = 12

46. The steady state error of the system shown in the figure for a unit step input is _______.

R (s) + E ( s) 1 C ( s)
K=4
r (t) − e (t) s+2 C(t)
2
s+4

Answer: 0.5
4 2
Exp: Given G ( s ) = ;H ( s ) =
s+2 s+4
For unit step input,

k p = lim G ( s ) H ( s )
s→0

 4  2 
k p = lim   
s→0 s + 2
  s + 4 

kp = 1

A
Steady state error ess =
1 + kp

1
ess =
1+1
1
ess = ⇒ 0.50
2

 India’s No.1 institute for GATE Training  1 Lakh+ Students trained till date  65+ Centers across India
27
EC-GATE-2014 PAPER-03| www.gateforum.com

47. The state equation of a second-order linear system is given by


x ( t ) = Ax ( t ) , x ( 0) = x 0

1  e− t  0  e − t − e −2 t 
For x 0 =   , x ( t ) =  − t  and for x 0 =   , x ( t ) =  − t −2 t 
 −1   −e  1   − e + 2e 
3
when x 0 =   , x ( t ) is
5
 −8e − t + 11e −2 t   11e − t − 8e −2t 
( A)  −t −2 t 
( B)  −t −2t 
 8e − 22e   −11e + 16e 
 3e − t − 5e −2t   5e − t − 3e −2 t 
(C)  −t −2t 
( D)  −t −2 t 
 −3e + 10e   −5e + 6e 
Answer: B
Exp: Apply linearity principle,
3   1 0
5 = a  −1 + b 1  s
     
a =3; b =8

 e− t  e − t − e −2 t 
⇒ x ( t ) = 3  −t  +  −t −2t 
 −e   −e + 2e 
 11e− t − 8e−2t 
⇒ x(t) =  −t −2 t 
 −11e + 16e 

48. In the root locus plot shown in the figure, the pole/zero marks and the arrows have been
removed. Which one of the following transfer functions has this root locus?

1 σ

s +1 s+4
( A) ( B)
( s + 2 )( s + 4)( s + 7 ) ( s + 1)( s + 2)( s + 7)
s+7 ( s + 1)( s + 2)
( C) ( D)
( s + 1)( s + 2)( s + 4) ( s + 7)( s + 4)

 India’s No.1 institute for GATE Training  1 Lakh+ Students trained till date  65+ Centers across India
28
EC-GATE-2014 PAPER-03| www.gateforum.com

Answer: B

Exp:: For transfer function


(s + 4)
( s + 1)( s + 2 )( s + 3)
From pole zero plot

σ
−7 −4 −2 −1

49. Let X(t) be a wide sense stationary (WSS) random process with power spectral density SX(f).
If Y(t) is the process defined as Y ( t ) = X ( 2t − 1) , the power spectral density SY(f) is
1 f  1 f 
(A) SY ( f ) = SX   e− jπf ( B) SY ( f ) = SX   e − jπf / 2
2 2 2 2
1 f  1 f 
(C) SY ( f ) = SX   ( D) SY ( f ) = SX   e − j2 πf
2 2 2 2
Answer: C
Exp: Shifting in time domain does not change PSD. Since PSD is Fourier transform of
autocorrelation function of WSS process, autocorrelation function depends on time
difference.
X ( t ) ↔ R x ( z ) ↔ Sx ( f )
1 f 
Y ( t ) = X ( 2t − 1) ↔ R y ( 2ζ ) ↔ Sx  
2 2
[time scaling property of Fourier transform]

50. A real band-limited random process X(t) has two-sided power spectral density

Sx (f ) = { 10 − 6 ( 3000 − f
0
) W atts / H z for f ≤ 3 kH z
otherw ise

Where f is the frequency expressed in Hz. The signal X(t)modulates a carrier cos16000 πt
and the resultant signal is passed through an ideal band-pass filter of unity gain with centre
frequency of 8 kHz and band-width of 2 kHz. The output power (in Watts) is _______.
Answer: 2.5
Exp:

Sx ( f ) 3 × 10−3 watts
Sx ( f )

−3 +3 f (in KHz )

 India’s No.1 institute for GATE Training  1 Lakh+ Students trained till date  65+ Centers across India
29
EC-GATE-2014 PAPER-03| www.gateforum.com

After modulation with cos (16000πt )


1
Sy ( f ) =Sx ( f − f c ) + Sx ( f + f c ) 
4
This is obtain the power spectral density Random process y(t), we shift the given power
spectral density random process x(t) to the right by fc shift it to be the left by fc and the two
shifted power spectral and divide by 4.

1.5 × 10 −3 / 2

−11 −8 −5 5 8 11 f (in KHz )

After band pass filter of center frequency 8 KHz and BW of 2 kHz

1.5 × 10 −3 / 2

−9 − 8 − 7 7 8 9

Total output power is area of shaded region


= 2 [ Area of one side portion ]
= 2 [ Area of triangle + Area of rec tan gle ]
 1 
2  − × 2 × 103 × 0.5 × 10−3 + 2 × 103 × 1 × 10−3 
=  
2
2
= [ 0.5 + 2] = 2.5 watts

51. In a PCM system, the signal m ( t ) = { sin (100πt ) + cos (100πt )} V is sampled at the Nyquist
rate. The samples are processed by a uniform quantizer with step size 0.75 V. The minimum
data rate of the PCM system in bits per second is _____.
Answer: 200
Exp: Nyquist rate = 2 × 50 Hz
= 100 samples / sec

∆=
m ( t ) max − m ( t ) min
⇒L=
(
2− − 2 )
L 0.75
2 2
L= = 3.77 = 4
0.75
No. of bits required to encode ‘4’ levels = 2 bits/level
Thus data rate = 2 × 100 = 200 bits / sec

 India’s No.1 institute for GATE Training  1 Lakh+ Students trained till date  65+ Centers across India
30
EC-GATE-2014 PAPER-03| www.gateforum.com

52. A binary random variable X takes the value of 1 with probability 1/3. X is input to a cascade
of 2 independent identical binary symmetric channels (BSCs) each with crossover probability
1/2. The outputs of BSCs are the random variables Y1 and Y2 as shown in the figure.

X Y1 Y2
BSC BSC

The value of H(Y1) + H(Y2) in bits is___________.


Answer: 2
1 2
Exp: Let P {x = 2} = , P {x = 0} =
3 3
to find H ( Y1 ) we need to know P { y1 = 0} and P { y 2 = 1}

P {Y1 = 0} = P {Y1 = 0 / x1 = 0} P {x1 = 0} + P { y1 = 0 / x1 = 1} P {x1 = 1}

1 1 1 2 1
= . + × =
2 3 2 3 2
1
P {y1 = 1} =
2
1 1
⇒ H ( y1 ) = log 2 2 + log 2 2 = 1
2 2
Similarly
1 1
P {y 2 = 0} = and P { y 2 = 1} =
2 2
⇒ H {y 2 } = 1

⇒ H {y1} + H {y 2 } = 2 bits

53. Given the vector A = ( cos x )( sin y ) aˆ x + ( sin x )( cos y ) aˆ y , where â x , â y denote unit vectors
along x,y directions, respectively. The magnitude of curl of A is ________
Answer: 0
Exp (1):
aˆ x aˆ y aˆ z
 ∂ ∂ ∂
Curl A =
∂x ∂y ∂z
cos x sin y sin x cos y 0

=0

∴ Curl A = 0

 India’s No.1 institute for GATE Training  1 Lakh+ Students trained till date  65+ Centers across India
31
EC-GATE-2014 PAPER-03| www.gateforum.com

Exp(2):
Given A = cos x sin yaˆ x + sin x cos y aˆ y
ax ay az
∇×A = ∂ / ∂x ∂ / ∂y ∂ / ∂z
cos x sin y sin x cos y 0
= a x ( 0 ) − a y ( 0 ) + a z ( cos x cos y − cos x cos y ) = 0
∴ ∇×A = 0

54. A region shown below contains a perfect conducting half-space and air. The surface current
  
K s on the surface of the perfect conductor is K s = x2
ˆ amperes per meter. The tangential H
field in the air just above the perfect conductor is
y


KS Air
x

Perfect conductor

(A) ( xˆ + zˆ ) 2 amperes per meter (B) x̂2 amperes per meter


(C) − ẑ2 amperes per meter (D) ẑ2 amperes per meter
Answer: D
Exp: Given medium (1) is perfect conductor
Medium (2) is air
∴ H1 = 0
From boundary conditions
( H1 − H 2 ) × a n = K S
H1 = 0
K S = 2aˆ x
an = ay
− H 2 × a y = 2aˆ x
− ( H x a x + H y a y + H z a z ) × a y = 2a x
− H x a z + H z a x = 2a x
∴ Hz = 2
H = 2a z

 India’s No.1 institute for GATE Training  1 Lakh+ Students trained till date  65+ Centers across India
32
EC-GATE-2014 PAPER-03| www.gateforum.com

( )

55. Assume that a plane wave in air with an electric field E = 10cos ωt − 3x − 3z aˆ y V/m is
incident on a non-magnetic dielectric slab of relative permittivity 3 which covers the region.
Z > 0 The angle of transmission in the dielectric slab is _________________ degrees.
Answer: 30
Exp: (
Given E = 10cos ωt − 3x − 3z a y )
(
− Jβ x cos θx + y cos θ y + z cos θz )
E = E 0e
So, β x = β cos θx = 3
β y = β cos θ y = 0
βz = β cos θz = 3
β x + β y 2 + βz 2 = β2
2

9 + 3 = β2 ⇒ β = 13
3
β cos θz = 3 ⇒ cos θz = ⇒ θz = 61.28 = θi
13
sin θi E2 sin 61.28 3 0.8769
= ⇒ = ⇒ = sin θt
sin θt E1 sin θt 1 3
θt = 30.4 ⇒ θt  30o

 India’s No.1 institute for GATE Training  1 Lakh+ Students trained till date  65+ Centers across India
33
EC-GATE-2014 PAPER-04| www.gateforum.com

Q. No. 1 – 5 Carry One Mark Each

1. Which of the following options is the closest in meaning to the word underlined in the
sentence
below? In a democracy, everybody has the freedom to disagree with the government.
(A) Dissent (B) Descent (C) Decent (D) Decadent
Answer: A
Exp: Dissent is to disagree

2. After the discussion, Tom said to me, 'Please revert!’ He expects me to _________.
(A) Retract (B) Get back to him
(C) Move in reverse (D) Retreat
Answer: B
Exp: Revert means set back

3. While receiving the award, the scientist said, "I feel vindicated". Which of the following is
closest in meaning to the word ‘vindicated’?
(A) Punished (B) Substantiated (C) Appreciated (D) Chastened
Answer: B
Exp: Vindicate has 2 meanings
1. Clear of blain
2. Substantiate, justify

4. Let f ( x, y ) = x n y m = P. If x is doubled and y is halved, the new value of f is

( A) 2n−m P ( B) 2 m− n P ( C) 2 ( n − m) P ( D) 2 ( m − n ) P
Answer: A
m
y
Exp: (2x) ×   = 2n − m × x n y m
n

2

5. In a sequence of 12 consecutive odd numbers, the sum of the first 5 numbers is 425. What is
the sum of the last 5 numbers in the sequence?
Answer: 495
Exp: Let consecutive odd numbers be a-10, a-8, a-6, a-4, a-2, a, ……a+12
Sum of 1st 5 number = 5a-30=425 ⇒ a=91
Last 5 numbers=(a+4)+(a+6)+…….+(a+12)
=(95+97+99+101+103)= 495

 India’s No.1 institute for GATE Training  1 Lakh+ Students trained till date  65+ Centers across India
1
EC-GATE-2014 PAPER-04| www.gateforum.com

Q. No. 6 – 10 Carry Two Marks Each

6. Find the next term in the sequence: 13M, 17Q, 19S, ___
(A) 21W (B) 21V (C) 23W (D) 23V
Answer: C
Exp:
13 M

17(13 + 4) Q(M + 4)
19(17 + 2) S(Q + 2)
23(19 + 4) W = (s+ 4)
⇒ 23W

7. If ‘KCLFTSB’ stands for ‘best of luck’ and ‘SHSWDG’ stands for ‘good wishes’, which of
the following indicates ‘ace the exam’?
(A) MCHTX (B) MXHTC (C) XMHCT (D) XMHTC
Answer: B
Exp: KCLFTSB SHSWDG
Reverse order: Reverse order:
BCS TOF LUCK GO OD W I S HES
Ace the exam
Reverse order should be
MAXE EHT ECA
Looking at the options we have M X H T C

8. Industrial consumption of power doubled from 2000-2001 to 2010-2011. Find the annual rate
of increase in percent assuming it to be uniform over the years.
(A) 5.6 (B) 7.2 (C) 10.0 (D) 12.2
Answer: B
Exp:
n
 r 
A = P 1 + 
 100 
A = 2P
10
 r 
2 = 1+ 
 100 
∴ r = 7.2

 India’s No.1 institute for GATE Training  1 Lakh+ Students trained till date  65+ Centers across India
2
EC-GATE-2014 PAPER-04| www.gateforum.com

9. A firm producing air purifiers sold 200 units in 2012. The following pie chart presents the
share of raw material, labour, energy, plant & machinery, and transportation costs in the total
manufacturing cost of the firm in 2012. The expenditure on labour in 2012 is Rs. 4,50,000. In
2013, the raw material expenses increased by 30% and all other expenses increased by 20%.
What is the percentage increase in total cost for the company in 2013?

Transpor Labour
− tation
30% 15%

Plant and Raw Material


machinery 20%
30%
Energy
25%

Answer: 22%
Exp: Let total cost in 2012 is 100
Raw material increases in 2013 to 1.3 x 20=26
Other Expenses increased in 2013 to 1.2 x 80=96
Total Cost in 2013 =96+26=122
Total Cost increased by 22%
Hint:Labour cost (i.e, 4,50,000) in 2012 is redundant data.

10. A five digit number is formed using the digits 1,3,5,7 and 9 without repeating any of them.
What is the sum of all such possible five digit numbers?
(A) 6666660 (B) 6666600 (C) 6666666 (D) 6666606
Answer: B
Exp: 1 appears in units place in 4! Ways
Similarly all other positions in 4! Ways
Same for other digits.
Sum of all the numbers = (11111) X 4! (1+3+5+7+9) = 6666600

 India’s No.1 institute for GATE Training  1 Lakh+ Students trained till date  65+ Centers across India
3
EC-GATE-2014 PAPER-04| www.gateforum.com

Q.No. 1 – 25 Carry One Mark Each


1
1. The series ∑ n! converges to
n =0

(A) 2 ln2 (B) 2 (C) 2 (D) e


Answer: D

1 1 1
Exp: ∑ n! = 1 + 1! + 2! + .........
n =0

x x2
= e as e x = 1 + + + ......., ∀ x in R
1! 2!

2. The magnitude of the gradient for the function f ( x, y, z ) = x 2 + 3y 2 + z 3 at the point (1,1,1) is
_________.
Answer: 7

( )(
  
Exp: ( ∇f )P(1,1,1) = i ( 2x ) + j ( 6y ) + k ( 3z 2 )
P 1,1,1)
  
= 2i + 6 j + 3k
( ∇f ) P = 4 + 36 + 9 = 7

3. Let X be a zero mean unit variance Gaussian random variable. E  X  is equal to _____
Answer: 0.8
1 − x2 2
Exp: X ~ N ( 0,1) ⇒ f ( x ) = e , −∞ < x < ∞

∴ E{ x } = ∫

x .f ( x ) dx
−∞

1 ∞ −x2
= x2 ∫ x e 2
dx
2π 0

2 ∞ 2
= ∫ e− u du = = 0.797  0.8
2π 0 π

4. If a and b are constants, the most general solution of the differential equation
d2x dx
2
+ 2 + x = 0 is
dt dt
( A ) ae − t ( B) ae− t + bte − t ( C) ae t + bte − t ( D ) ae −2t
Answer: B
A.E : − m 2 + 2m + 1 = 0 ⇒ m = −1, −1
Exp:
∴ general solution is x = ( a + bt ) e − t

 India’s No.1 institute for GATE Training  1 Lakh+ Students trained till date  65+ Centers across India
4
EC-GATE-2014 PAPER-04| www.gateforum.com

xy
5. The directional derivative of f ( x, y ) = ( x + y ) at (1,1) in the direction of the unit vector at
2
π
an angle of with y-axis, is given by _____ .
4
Answer: 3
  2xy + y 2    x 2 + 2xy 
Exp: f =
1
( x 2 y + xy 2 ) ⇒ ∇f = i   + j 
2  2   2 
3  3 
at (1,1) , ∇f = i+ j
2 2
π
ê = unit vector in the direction i.e., making an angle of with y-axis
4
 π   π 
=  sin  i +  cos  j
 4  4
 3  1 
∴ directional derivative = e.
ˆ ∇f = 2   =3
 2  2

6. The circuit shown in the figure represents a

Ii A l Ii R

(A) Voltage controlled voltage source (B) Voltage controlled current source
(C) Current controlled current source (D) Current controlled voltage source
Answer: C
Exp:

A1I1

The dependent source represents a current controlled current source

7. The magnitude of current (in mA) through the resistor R2 in the figure shown is_______.
R2

1kΩ
R1 R3
10 mA 4 kΩ 2 mA
2 kΩ

R 4 3kΩ

 India’s No.1 institute for GATE Training  1 Lakh+ Students trained till date  65+ Centers across India
5
EC-GATE-2014 PAPER-04| www.gateforum.com

Answer: 2.8
Exp: By source transformation
R 2 = 1kΩ 4 kΩ

I
2 kΩ
+ −
20V − + 8V

3kΩ

By KVL,
20 − 10k.I + 8 = 0
28
⇒I=
10 k
⇒ I = 2.8mA

8. At T = 300 K, the band gap and the intrinsic carrier concentration of GaAs are 1.42 eV and
106 cm-3, respectively. In order to generate electron hole pairs in GaAs, which one of the
wavelength ( λ C ) ranges of incident radiation, is most suitable? (Given that: Plank’s constant
is 6.62 × 10-34 J-s, velocity of light is 3 × 1010 cm/s and charge of electron is 1.6 × 10-19 C)
(A) 0.42 µm < λ C < 0.87 µm (B) 0.87 µm < λ C < 1.42 µm
(C) 1.42 µm < λ C < 1.62 µm (D) 1.62 µm < λ C < 6.62 µm
Answer: A
hC 6.62 × 10−34 × 3 × 108
Exp: E = ⇒λ= = 0.87 µm
λ 1.42 × 1.6 × 10−19

9. In the figure ln ( ρi ) is plotted as a function of 1/T, where ρi the intrinsic resistivity of silicon,
T is is the temperature, and the plot is almost linear.
ln ( ρi )

1/ T
The slope of the line can be used to estimate
(A) Band gap energy of silicon (Eg)
(B) Sum of electron and hole mobility in silicon µ n + µ p ( )
( )
−1
(C) Reciprocal of the sum of electron and hole mobility in silicon µ n + µ p

(D) Intrinsic carrier concentration of silicon ( n i )

 India’s No.1 institute for GATE Training  1 Lakh+ Students trained till date  65+ Centers across India
6
EC-GATE-2014 PAPER-04| www.gateforum.com

Answer: A
3
n i α T 2 e − Eg /kT and
Exp: 1
ρι α
ηi
∴ From the graph, Energy graph of Si can be estimated

10. The cut-off wavelength (in µm) of light that can be used for intrinsic excitation of a
semiconductor material of bandgap Eg= 1.1 eV is ________
Answer: 1.125
hC
Exp: E=
λ
6.6 × 10−34 × 3 × 108
⇒λ= = 1.125 µm
1.1 × 1.6 × 10−19

11. If the emitter resistance in a common-emitter voltage amplifier is not bypassed, it will
(A) Reduce both the voltage gain and the input impedance
(B) Reduce the voltage gain and increase the input impedance
(C) Increase the voltage gain and reduce the input impedance
(D) Increase both the voltage gain and the input impedance
Answer: B
Exp: When a CE amplifier’s emitter resistance is not by passed, due to the negative feedback the
voltage gain decreases and input impedance increases

12. Two silicon diodes, with a forward voltage drop of 0.7 V, are used in the circuit shown in the
figure. The range of input voltage Vi for which the output voltage V0 = Vi , is
R
+ +

D1 D2
Vi VO

−1V ± ± 2V
− −

( A ) − 0.3V < Vi < 1.3V ( B) − 0.3V < Vi < 2V


( C) − 1.0 V < Vi < 2.0V ( D ) − 1.7 V < Vi < 2.7V
Answer: D
Exp: When Vi < −1.7V ; D1 − ON and D 2 − OFF
∴ V0 = −1.7V
When Vi > 2.7V; D1 − OFF & D 2 − ON
∴ V0 = 2.7V
When − 1.7 < Vi < 2.7V, Both D1 & D 2 OFF
∴ V0 = Vi

 India’s No.1 institute for GATE Training  1 Lakh+ Students trained till date  65+ Centers across India
7
EC-GATE-2014 PAPER-04| www.gateforum.com

13. The circuit shown represents:

C2 + 12 V
vi −
v0
R2 + − 12 V

R1
−2 V C1

(A) A bandpass filter (B) A voltage controlled oscillator


(C) An amplitude modulator (D) A monostable multivibrator
Answer: D

14. For a given sample-and-hold circuit, if the value of the hold capacitor is increased, then
(A) Droop rate decreases and acquisition time decreases
(B) Droop rate decreases and acquisition time increases
(C) Droop rate increases and acquisition time decreases
(D) Droop rate increases and acquisition time increases
Answer: B
dv
Exp: Capacitor drop rate =
dt
dv 1
For a capacitor, ∝
dt c
∴ Drop rate decreases as capacitor value is increased
For a capacitor, Q = cv = i × t ⇒ t ∝ c
∴ Acquisition time increases as capacitor value increased

15. In the circuit shown in the figure, if C=0, the expression for Y is

C
A
B
Y

A
B

(A) Y = A B + A B ( B) Y = A + B
( C) Y = A + B ( D) Y = A B

 India’s No.1 institute for GATE Training  1 Lakh+ Students trained till date  65+ Centers across India
8
EC-GATE-2014 PAPER-04| www.gateforum.com

Answer: A
Exp: C=0 1

A A⋅B
B
A⋅B Y

A ⋅ B + A ⋅ B = AB
A
B A⋅B

Y = 1.A  B
=A  B
= A ⊕ B = A B + AB + AB

16. The output (Y) of the circuit shown in the figure is

VD0
( A) A+ B+C

A B C

( B) A + B . C + A .C
A Output ( Y )

B
( C) A+ B+ C

( D) A.B .C
Answer: A
Exp: VDD

A B C

output ( Y )
A

 India’s No.1 institute for GATE Training  1 Lakh+ Students trained till date  65+ Centers across India
9
EC-GATE-2014 PAPER-04| www.gateforum.com

This circuit is CMOS implementation


If the NMOS is connected in series, then the output expression is product of each input with
complement to the final product.
So, Y = A.B .C
= A + B +C

17. A Fourier transform pair is given by


n − j6 πf
2 FT Ae
  u [ n + 3] ⇔
 
3 2
1 −   e − j2 πf
3
where u[n] denotes the unit step sequence. The value of A is _________.
Answer: 3.375
n
2
Exp: x [ n ] =   u [ n + 3]
3
−3
2 j3Ω
∞ n   .e
X ( e jΩ ) =
2  3
∑  3  .e − jΩn =
2
n =−3
1 − e − jΩ
3
3
 3  27
⇒A=  = = 3.375
2 8

w
18. A real-valued signal x(t) limited to the frequency band f ≤ is passed through a linear time
2
invariant system whose frequency response is
 − j4 πf w
e , f ≤
H (f ) = 
2
 0, f > w
 2
The output of the system is
( A ) x ( t + 4) ( B) x ( t − 4 ) ( C) x ( t + 2) ( D) x ( t − 2)
Answer: D
Exp: Let x ( t ) Fourier transform be x ( t )

x (t )
h (t ) y (t )

y ( t ) = x ( t ) * h ( t ) [ convolution ]
⇒ Y ( f ) = X ( f ) .H ( f )
⇒ Y ( f ) = e − j4 πf .X ( f )
⇒ y ( t ) = x ( t − 2)

 India’s No.1 institute for GATE Training  1 Lakh+ Students trained till date  65+ Centers across India
10
EC-GATE-2014 PAPER-04| www.gateforum.com

19. The sequence x[n] = 0.5n u[n], where u[n] is the unit step sequence, is convolved with itself to

obtain y[n]. Then ∑ y ( n ) _________________.
n =−∞

Answer: 4
Exp: y [n ] = x [n ] * x [n ]

Let Y ( e jΩ ) is F.T. pair with y [ n ]


⇒ Y ( e jΩ ) = X ( e jΩ ) .X ( e jΩ )

Y ( e jΩ ) =
1 1
− jΩ
.
1 − 0.5e 1 − 0.5e − jΩ

also Y ( e jΩ ) = ∑ y [ n ].e − jΩn

h =−∞

∑ y [ n ] = Y ( e ) = 0.5 . 0.5 = 4
1 1
⇒ j0

n =−∞

20. In a Bode magnitude plot, which one of the following slopes would be exhibited at high
frequencies by a 4th order all-pole system?
(A) – 80 dB/decade (B) – 40 dB/decade
(C) +40 dB/decade (D) +80 dB/decade
Answer: A
Exp: → In a BODE diagram, in plotting the magnitude with respect to frequency, a pole introduce a
line 4 slope −20dB / dc
→ If 4th order all-pole system means gives a slope of ( −20 ) * 4 dB / dec i.e. − 80dB / dec

21. For the second order closed-loop system shown in the figure, the natural frequency (in rad/s)
is
U (s) + 4 Y (s)
S (S + 4 )

(A) 16 (B) 4 (C) 2 (D) 1


Answer: C
Y (s) 4
Exp: Transfer function =
U (s) S + 4s + 4
2

If we compare with standard 2nd order system transfer function


wn2
i.e.,
s 2 + 2ξw n s + w n 2
w n 2 = 4 ⇒ w n = 2 rad / sec

 India’s No.1 institute for GATE Training  1 Lakh+ Students trained till date  65+ Centers across India
11
EC-GATE-2014 PAPER-04| www.gateforum.com

22. If calls arrive at a telephone exchange such that the time of arrival of any call is independent
of the time of arrival of earlier or future calls, the probability distribution function of the total
number of calls in a fixed time interval will be
(A) Poisson (B) Gaussian (C) Exponential (D) Gamma
Answer: A
Exp: Poisson distribution: It is the property of Poisson distribution.

23. In a double side-band (DSB) full carrier AM transmission system, if the modulation index is
doubled, then the ratio of total sideband power to the carrier power increases by a factor of
_________________.
Answer: 4
Ratio of total side band power
Exp: α µ2
Carrier power
If it in doubled, this ratio will be come 4 times
24. For an antenna radiating in free space, the electric field at a distance of 1 km is found to be 12
mV/m. Given that intrinsic impedance of the free space is 120 π Ω, the magnitude of average
power density due to this antenna at a distance of 2 km from the antenna (in nW/m2 )
is________________.
Answer: 47.7
Exp: Electric field of an antenna is
 
 
ηI0 dl  Jβ 1 J 
Eθ = sin θ  + 2 −
4π r r βr 3 
 ↓ ↓ ↓ 
 field
Radiation inductive Electrostatic

 field field 
1
∴E α
r
E1 r2
= ⇒ E 2 = 6 mv / m
E 2 r1
1 E 2 1 36 × 10−8
P= = = 47.7 nw / m 2
2 η 2 120π

25. Match column A with column B.


Column A Column B
(1) Point electromagnetic source (P) Highly directional
(2) Dish antenna (Q) End free
(3) Yagi-Uda antenna (R) Isotropic
1→ P 1→ R 1→ Q 1→ R
(A) 2 → Q (B) 2 → P (C) 2 → P (D) 2 → Q
3→ R 3→ Q 3→ R 3→ P

 India’s No.1 institute for GATE Training  1 Lakh+ Students trained till date  65+ Centers across India
12
EC-GATE-2014 PAPER-04| www.gateforum.com

Answer: B
Exp: 1. Point electromagnetic source, can radiate fields in all directions equally, so isotropic.
2. Dish antenna → highly directional
3. Yagi – uda antenna → End fire

R F

Figure: Yagi-uda antenna

Q. No. 26 – 55 Carry Two Marks Each

26. With initial values y(0) = y’(0)=1 the solution of the differential equation
d2y dy
2
+ 4 + 4y = 0
dx dx
at x = 1 is ________
Answer: 0.54
Exp: A.E : m 2 + 4m + 4 = 0 ⇒ m = −2, −2
∴ solutions is y = ( a + bx ) e −2x .......(1)
y ' = ( a + bx ) ( −2e −2 x ) + e −2 x ( b ) ........ ( 2 )
u sin g y ( 0 ) = 1; y ( 0 ) = 1, (1) and ( 2 ) gives
'

a = 1 and b = 3
∴ y = (1 + 3x ) e−2x
at x = 1, y = 4e −2 = 0.541  0.54

27. Parcels from sender S to receiver R pass sequentially through two post-offices. Each post-
1
office has a probability of losing an incoming parcel, independently of all other parcels.
5
Given that a parcel is lost, the probability that it was lost by the second post-office
is_________.
Answer: 0.44
Exp: Parcel will be lost if
a. it is lost by the first post office
b. it is passed by first post office but lost by the second post office
1 4 1 9
Prob(parcel is lost) = + x =
5 5 5 25
P (Parcel lost by second post if it passes first post office)= P (Parcel passed by first post
office) x P(Parcel lost by second post office)

 India’s No.1 institute for GATE Training  1 Lakh+ Students trained till date  65+ Centers across India
13
EC-GATE-2014 PAPER-04| www.gateforum.com

4 1 4
= × =
5 5 25
4 / 25 4
Prob(parcel lost by 2nd post office | parcel lost)= = = 0.44
9 / 25 9

1
28. The unilateral Laplace transform of f ( t ) is . Which one of the following is the
s + s +1
2

unilateral Laplace transform of g(t) = t. f(t)?


−s − ( 2s + 1) S 2S + 1
( A) ( B) ( C) ( D)
(s ) (s ) (s ) (s )
2 2 2 2
2
+ s +1 2
+ s +1 2
+ s +1 2
+ s +1
Answer: D
Exp: (1)
If f ( t ) ↔ F ( s )
−d
Then tf ( t ) ↔ F(s)
ds
−d  1 
=  2 
ds  s + s + 1 
− ( 2s + 1) 2s + 1
=− =
( s2 + s + 1) ( s2 + s + 1)
2 2

Exp: (2)
1
F (s ) =
s + s +12

d
L  g ( t ) = t.f ( t )  = −  F ( s )  ( u sin g multiplication by t )
ds 
2s + 1
=
(s + s + 1)
2 2

29. For a right angled triangle, if the sum of the lengths of the hypotenuse and a side is kept
constant, in order to have maximum area of the triangle, the angle between the hypotenuse
and the side is
(A) 12O (B) 36O (C) 60O (D) 45O
Answer: (C) ( As per IIT Website)
Exp: Let x (opposite side), y (adjacent side) and z (hypotenuse) of a right angled triangle.
Given Z + y = K ( cons tan t ) ......(1) and angle between them say ' θ ' then Area,

1 1 z2
A= xy = ( z sin θ )( z cos θ ) = sin 2θ
2 2 4
k
Now (1) ⇒ z + z sin θ = k ⇒ z =
1 + sin θ

 India’s No.1 institute for GATE Training  1 Lakh+ Students trained till date  65+ Centers across India
14
EC-GATE-2014 PAPER-04| www.gateforum.com

k2  sin 2θ 
∴A =  
 (1 + sin θ ) 
2
4
dA
In order to have max imum area, =0

k 2  (1 + sin θ ) ( 2cos 2θ ) − sin 2θ ( cos θ ) .2 (1 + sin θ ) 
2

⇒  =0
(1 + sin θ )
4
4  
π
⇒θ= = 30o , Answer obtained is different than official key
6

30. The steady state output of the circuit shown in the figure is given by
y ( t ) = A ( ω) sin ( ωt + φ ( ω) ) . If the amplitude A ( ω ) = 0.25, then the frequency ω is

R C y (t)
± sin ( ωt ) C
C

1 2 1 2
( A) ( B) ( C) ( D)
3RC 3RC RC RC
Answer: B
Exp: V

y (t )
R C
+
− sin ωt C
C

V − 1 0o V V
+ + =0
( ) ( )
By nodal method,
R 1 2
j ωc j ωc
1 j ωc  1 0 o
V  + j ωc + =
R 2  R
2
V=
2 + 3jωRC
V 1
Y= ⇒
2 2 + jω3RC
1 1
given A ( ω) = ⇒
4 4 + 9R 2 c2 .ω2
2
⇒ ω=
3 RC

 India’s No.1 institute for GATE Training  1 Lakh+ Students trained till date  65+ Centers across India
15
EC-GATE-2014 PAPER-04| www.gateforum.com

31. In the circuit shown in the figure, the value of V0(t) (in Volts) for t → ∞ is ______.
ix
2H

+ 2i +
10 u ( t ) A − x 5Ω Vo ( t )

5Ω

Answer: 31.25
Exp: B ix

2H
+ +
− 2ix
10x ( t ) 5Ω V0 ( t )
A 5Ω −

For t → ∞ , i.e., at steady state, inductor will behave as a shot circuit and hence VB = 5.i X
50
By KCL at node B, − 10 + VB − 2i x + i x = 0 ⇒ i x =
8
250
V0 ( t ) = 5i x ( t ) ⇒ V0 ( t ) = = 31.25 volts
8

32. The equivalent resistance in the infinite ladder network shown in the figure is Re.

2R R R R

Re R R R R

The value of Re/R is ________


Answer: 2.618
Exp:
R R R R

R R R R

R e qu

→ For an infinite ladder network, if all resistance are having same value of R
1+ 5 
Then equivalent resistance is   .R
 2 
→ For the given network, we can split in to R is in series with Requivalent

 India’s No.1 institute for GATE Training  1 Lakh+ Students trained till date  65+ Centers across India
16
EC-GATE-2014 PAPER-04| www.gateforum.com

Re q =
(1 + 5 ) R
2

R e qu

R equ
⇒ R equ = R + 1.618R ⇒ = 2.618
R

33. For the two-port network shown in the figure, the impedance (Z) matrix (in Ω ) is

30 Ω
1 2
+ +
10 Ω 60 Ω
− −
1' 2'

 6 24  9 8  9 6   42 6 
( A)  42 9  ( B) 8 24  ( C) 6 24  ( D)  6 60
       
Answer: C
Exp: For the two-part network
1 1 1 
 30 + 10 −
30 
Y matrix =  
 −1 1
+
1
 30 60 30 
Zmatrix = [ Y ]
−1

−1
 0.1333 − 0.0333
Z=
 −0.0333 0.05 
9 6 
Z= 
 6 24 

34. Consider a silicon sample doped with ND = 1×1015/cm3 donor atoms. Assume that the
intrinsic carrier concentration ni = 1.5×1010/cm3. If the sample is additionally doped with NA
= 1×1018/cm3 acceptor atoms, the approximate number of electrons/cm3 in the sample, at
T=300 K, will be _________________.
Answer: 225.2
Exp: P = N A − N D = 1 × 1018 − 1 × 1015 = 9.99 × 1017

η 2 (1.5 × 10 )
10 2

η= i = = 225.2 / cm3
P 9.99 × 1017

 India’s No.1 institute for GATE Training  1 Lakh+ Students trained till date  65+ Centers across India
17
EC-GATE-2014 PAPER-04| www.gateforum.com

35. Consider two BJTs biased at the same collector current with area A1 = 0.2µm × 0.2 µm and
A 2 = 300 µm × 300 µm . Assuming that all other device parameters are identical, kT/q = 26
mV, the intrinsic carrier concentration is 1 × 1010 cm-3, and q = 1.6 × 10-19 C, the difference
between the base-emitter voltages (in mV) of the two BJTs (i.e., VBE1 – VBE2) is
_________________.
Answer: 381
Exp: IC1 = IC2 ( Given )
VBE1

IS1 e VT = IS2 e BE2


V /VT

( VBE1 − VBE2 ) IS
e VT
= 2
IS1
IS2  300 × 300 
VBE1 − VBE 2 = VT ln = 26 × 10−3 ln   ∵ I S αA
IS1  0.2 × 0.2 
(V BE1 )
− VBE2 = 381mV

36. An N-type semiconductor having uniform doping is biased as shown in the figure.
V

N − type semiconductor

If EC is the lowest energy level of the conduction band, EV is the highest energy level of the
valance band and EF is the Fermi level, which one of the following represents the energy
band diagram for the biased N-type semiconductor?
EC EC
EF
( A) ( B) EF

EV EV

EC
EF
( D)
( C)
EV EC
EF
EV
Answer: D

37. Consider the common-collector amplifier in the figure (bias circuitry ensures that the
transistor operates in forward active region, but has been omitted for simplicity). Let IC be the
collector current, VBE be the base-emitter voltage and VT be the thermal voltage. Also, g m
and r0 are the small-signal transconductance and output resistance of the transistor,
respectively. Which one of the following conditions ensures a nearly constant small signal
voltage gain for a wide range of values of RE?

 India’s No.1 institute for GATE Training  1 Lakh+ Students trained till date  65+ Centers across India
18
EC-GATE-2014 PAPER-04| www.gateforum.com

VCC

Vin Vout

RE

( A) g m R E << 1 ( B) I C R E >> VT ( C) g m r0 >> 1 ( D) VBE >> Vr


Answer: B
RE RE IE R E
Exp: AV = = =
re + R E VT
+ R E VT + I E R E
IE
IC R E
∴ AV  (∵ IC  IE )
VT + IC R E
∴ IC R E >> U T ⇒ A V in almost cons tan t

38. A BJT in a common-base configuration is used to amplify a signal received by a 50Ω


antenna. Assume kT/q = 25 mV. The value of the collector bias current (in mA) required to
match the input impedance of the amplifier to the impedance of the antenna is________.
Answer: 0.5
VI
Exp: Input impedance of CB amplifier, z i = re =
IE
25 mV
⇒ 50 = (∵ signal is received from 50Ω antenna and VT = 25 mV )
IE
25mV
⇒ IE = = 0.5 mA
50Ω

39 . For the common collector amplifier shown in the figure, the BJT has high β , negligible
VCE(sat), and VBE = 0.7 V. The maximum undistorted peak-to-peak output voltage vo (in Volts)
is______.

VCC = +12V

R1
5kΩ
1 µF
vi
1 µF
R2 vo
10 kΩ RE
1kΩ

 India’s No.1 institute for GATE Training  1 Lakh+ Students trained till date  65+ Centers across India
19
EC-GATE-2014 PAPER-04| www.gateforum.com

Answer: 9.4
Exp: ∵ β = high, I B is neglected
10 k
∴ VB = 12 × = 8V
10 k + 5k
VE = VB − 0.7 = 7.3V
∴ VCE = 12 − 7.3 = 4.7V
∴ Maximum undistorted V0 ( p − p ) = 2 × 4.7V = 9.4V

40. An 8-to-1 multiplexer is used to implement a logical function Y as shown in the figure. The
output Y is given by

0 I0
D I1
0 I2
D I3
0 I4 Y
0 I5
1 I6
0 I7 S2 S1 S0

A B C

( A) Y= A BC + ACD ( B) Y= A BC + A BD

( C) Y= A BC + ACD ( D) Y= A BD + A BC
Answer: C
Exp: Y = ABCD + ABCD + ABC 0 Io
D I1
I2
Remaining combinations of the select 0
D I3 8 :1 Y
lines will produce output 0. 0 I4
MUX
I5
So, Y = ACD ( B + B ) + ABC
0
1 I6
0 I7
S 2 S1 S0
= ACD + ABC
= ABC + ACD B C
A

41. A 16-bit ripple carry adder is realized using 16 identical full adders (FA) as shown in the
figure. The carry-propagation delay of each FA is 12 ns and the sum-propagation delay of
each FA is 15 ns. The worst case delay (in ns) of this 16-bit adder will be __________.
A0 B0 A1 B1 A14 B14 A15 B15

C0 C1 C14 C15
FA 0 FA1 FA14 FA15

S0 S1 S14 S15

 India’s No.1 institute for GATE Training  1 Lakh+ Students trained till date  65+ Centers across India
20
EC-GATE-2014 PAPER-04| www.gateforum.com

Answer: 195
Exp: A1 A14 A15
A0 B0 B1 B14 B15

C1 C14 C15
FA 0
C0
FA1 ........ . FA14 FA15

S0 S1 S14 S15

This is 16-bit ripple carry adder circuit, in their operation carry signal is propagating from 1st
stage FA0 to last state FA15, so their propagation delay is added together but sum result is
not propagating. We can say that next stage sum result depends upon previous carry.
So, last stage carry (C15) will be produced after 16 ×12 ns = 192 ns
Second last stage carry (C14) will be produced after 180 ns.
For last stage sum result (S15) total delay = 180ns + 15ns = 195ns
So, worst case delay = 195 ns

42. An 8085 microprocessor executes “STA 1234H” with starting address location 1FFEH (STA
copies the contents of the Accumulator to the 16-bit address location). While the instruction
is fetched and executed, the sequence of values written at the address pins A15-A8 is
(A) 1FH, 1FH, 20H, 12H (B) 1FH, FEH, 1FH, FFH, 12H
(C) 1FH, 1FH, 12H, 12H (D) 1FH, 1FH, 12H, 20H, 12H
Answer: A
Exp: Let the opcode of STA is XXH and content of accumulator is YYH.
Instruction: STA 1234 H
Starting address given = 1FFEH
So, the sequence of data and addresses is given below:

Address (in hex) : Data (in hex)


A 15 − A 8 A 7 − A 0

1F FE H → XXH
1F FF H → 34H
20 00 H → 12 H
12 34 H → YYH

1
43. A stable linear time invariant (LTI) system has a transfer function H ( s) = . To make
s2 + s − 6
this system causal it needs to be cascaded with another LTI system having a transfer function
H1(s). A correct choice for H1(s) among the following options is
(A) s + 3 (B) s - 2 (C) s - 6 (D) s + 1

 India’s No.1 institute for GATE Training  1 Lakh+ Students trained till date  65+ Centers across India
21
EC-GATE-2014 PAPER-04| www.gateforum.com

Answer: B
1 1
Exp: Given, H ( s ) = =
s + s − 6 ( s + 3)( s − 2 )
2

It is given that system is stable thus its ROC includes jω axis . This implies it cannot be
causal, because for causal system ROC is right side of the rightmost pole.
⇒ Poles at s = 2 must be removes so that it can be become causal and stable
simultaneously.
1 1
⇒ ( s − 2) =
( s + 3)( s − 2) s+3

Thus H1 ( s ) = s − 2

44. A causal LTI system has zero initial conditions and impulses response h(t). Its input x(t) and
output y(t) are related through the linear constant-coefficient differential equation
d2 y ( t ) dy ( t )
2
+a + a2y ( t) = x ( t)
dt dt
Let another signal g(t) be defined as
dh ( t )
g ( t ) = a 2 ∫ h ( τ ) dτ + + ah ( t )
t

0 dt
If G(s) is the Laplace transform of g(t), then the number of poles of G(s) is _______.
Answer: 1
Exp: Given differential equation
s 2 y ( s ) + α sy ( s ) + α 2 y ( s ) = x ( s )

y (s ) 1
⇒ = = H (s )
x (s ) s + αs + α 2
2

t
d
g ( t ) = α 2 ∫ h ( z ) dz + h ( t ) + αh ( t )
0 dt

H (s)
= α2 + SH ( s ) + α H ( s )
s
1 1 α
= α2 +s 2 + 2
s ( s + αs + α )
22
( s + 2s + α )
2
s + α s + α2

α 2 + αs + s 2 1
= =
s ( s + αs + α ) s
2 2

No. of poles = 1

 India’s No.1 institute for GATE Training  1 Lakh+ Students trained till date  65+ Centers across India
22
EC-GATE-2014 PAPER-04| www.gateforum.com

45. The N-point DFT X of a sequence x [ n ] ,0 ≤ n ≤ N − 1 is given by


N −1 2π
1 −j
X[k] = ∑ x [ n]e
nk,
N
0≤ k ≤N − L
N n=0

Denote this relation as X = DFT(x). For N = 4, which one of the following sequences
satisfies DFT(DFT (x))=x ?
( A) x = [1 2 3 4 ] ( B) x = [1 2 3 2 ]
( C) x = [1 3 2 2 ] ( D) x = [1 2 2 3]
Answer: B
Exp: This can be solve by directly using option and satisfying the condition given in question
X = DFT ( x )
N −1 2π
D FT ( D FT ( x ) ) = DFT ( X ) =
1 −j
∑ X [n ] e
nk
N

N n =0

DFT y [1 2 3 4]

1 1 1 1  1   10 
1 − j − 1 j   2   
X=
1     = 1  2 + 2 j
4 1 − 1 1 − 1  3  4 2 
    
1 + j − 1 − j  4   −2 − j2 
DFT of ( x ) will not result in [1 2 3 4]

Try with DFT of y [1 2 3 2]

1 1 1 1  1   8  4 
1 − j − 1 j   2     
X=
1     = 1  −2  =  −1
4 1 − 1 1 − 1  3  4  0  0 
      
1 + j − 1 − j  2   −2   −1

 4 1 1 1   4
1  2  1 
−   − j − 1 j   −1 1  4   2 
  
  1 1
DFT of = = =
 0 4 1 − 1 1 − 1   0  2  6  3 
        
 −1 1 + j − 1 − j  −1 4 2
Same as x
Then ‘B’ is right option

. 
x1  0 1   x1 
46. The state transition matrix φ ( t ) of a system  .  = 
  0 0  x 2 
x 2 
 t 1 1 0 0 1 1 t 
( A)   ( B)   ( C)   ( D)  
1 0  t 1 1 t  0 1

 India’s No.1 institute for GATE Training  1 Lakh+ Students trained till date  65+ Centers across India
23
EC-GATE-2014 PAPER-04| www.gateforum.com

Answer: D
Exp: Given state model,
 x 1 ( t )   0 1   x1 ( t ) 
 =  
 x 2 ( t )   0 0   x 2 ( t ) 
0 1 
A= 
0 0
φ ( t ) ⇒ state transistion matrix
φ ( t ) = L−1 ( SI − A ) 
−1
 
−1
s − 1 1 s 1
[SI − A ] = 
−1
 ⇒ 2 0
0 s  s  s 
1 1 2
s
φ ( t ) = L−1  s 
0 1 
 s 
1 t 
φ( t ) =  
0 1 

ps2 + 3ps − 2
47. Consider a transfer function G p ( s ) = with p a positive real parameter.
s2 + ( 3 + p ) s + ( 2 − p )
The maximum value of p until which GP remains stable is ________.
Answer: 2
ps 2 + 3ps − 2
Exp: Given G p ( s ) =
s2 + ( 3 + p ) s + ( 2 − p )
By R − H criteria
The characteristic equation is s 2 + ( 3 + p ) s + ( 2 − p ) = 0
i.e. s 2 + ( 3 + p ) s + ( 2 − p ) = 0
By forming R-H array,

s2 1 (2 − p)
s1 ( 3 + φ ) 0
s0 ( 2 − p )

For stability, first column elements must be positive and non-zero


i.e. (1)( 3 + p ) > 0 ⇒ p > −3
and ( 2 )( 2 − p ) > 0 ⇒ p < 2
i.e. −3 < p < 2

The maximum value of p unit which G p remains stable is 2

 India’s No.1 institute for GATE Training  1 Lakh+ Students trained till date  65+ Centers across India
24
EC-GATE-2014 PAPER-04| www.gateforum.com

48. The characteristic equation of a unity negative feedback system 1 + KG(s) = 0. The open loop
transfer function G(s) has one pole at 0 and two poles at -1. The root locus of the system for
varying K is shown in the figure.

ζ = 0.5
A

x x
( 0,0) σ
−1 1
O

3

The constant damping ratio line, for ζ = 0.5 , intersects the root locus at point A. The distance
from the origin to point A is given as 0.5. The value of K at point A is ________ .
Answer: 0.375
Exp: We know that the co-ordinate of point A of the given root locus i.e., magnitude condition
G (s) H (s) = 1
Here, the damping factor ξ = 0.5 and the length of 0A = 5
ξ = 0.5
A
Then in the right angle triangle *
OX OX 1
cos θ = ⇒ cos 60 = ⇒ OX = θ
OA 0.5 4 * * *O
−1 −2 −1
X
3 3
AX AX 3
⇒ sin θ = ⇒ sin 60 = ⇒ AX =
OA 0.5 4

So, the co-ordinate of point A is −1 + j 3


4 4
Substituting the above value of A in the transfer function and equating to 1
i.e. by magnitude condition,

k
=1
s ( s + 1)
2
s = −1 + j 3
4 4

2
1 3  9 3 
k= + . + 
16 16  16 16 

k = 0.375

 India’s No.1 institute for GATE Training  1 Lakh+ Students trained till date  65+ Centers across India
25
EC-GATE-2014 PAPER-04| www.gateforum.com

49. Consider a communication scheme where the binary valued signal X satisfies P{X = +
1}=0.75 and P{X = -1}= 0.25. The received signal Y = X + Z, where Z is a Gaussian random
variable with zero mean and variance σ 2 . The received signal Y is fed to the threshold
detector. The output of the threshold detector X̂ is:

X̂ = { +−1.1. Y>τ
Y ≤ τ.

ˆ ≠ X , the threshold τ should be


To achieve a minimum probability of error P X { }
(A) Strictly positive
(B) Zero
(C) Strictly negative
(D) Strictly positive, zero, or strictly negative depending on the nonzero value of σ 2
Answer: C
Exp: C

H1 : x = +1; H 0 : x = −1
P ( H1 ) = 0.75; P ( H 0 ) = 0.25
Received signal γ =X+Z
1
Where Z ∼ N ( 0, −2 ) ; f Z ( z ) = e− Z 2 σ2
2

σ 2π
 1+ Z if X = 1
Received signal γ = 
 −1 + Z if X = −1
1
1 − ( γ−1)2
f γ ( y H1 ) = e 2 σ2

σ 2π
1
1 − ( γ+1)2
f γ ( y H0 ) = e 2 σ2

σ 2π

At optimum threshold yopt: for minimum probability of error

f γ ( y H1 ) P ( H0 )
=
f γ ( y H0 ) P ( H1 )
y = yopt


1 
2 σ 2 
( γ−1)2 − ( γ+1)2  P ( H0 )
e =
yopt
P ( H1 )

+ 2 yopt σ2 P ( H0 )
e =
P ( H1 )
σ 2  P ( H 0 )  −1.1σ 2
y opt = ln   = = −0.55σ 2
2  P ( H1 )  2
y opt = Optimum threshold
y opt < 0 ∴ Threshold is negative.

 India’s No.1 institute for GATE Training  1 Lakh+ Students trained till date  65+ Centers across India
26
EC-GATE-2014 PAPER-04| www.gateforum.com

50. Consider the Z-channel given in the figure. The input is 0 or 1 with equal probability.

1.0
0 0

INPUT 0.25 OUTPUT

1 1
0.75

If the output is 0, the probability that the input is also 0 equals ______________
Answer: 0.8
Exp: Given channel
1.0
X=0 Y =0

input output
0.25

X =1 Y =1
0.75

We have to det er min e, P {x = 0 / y = 0}

P { y = 0 / x = 0} P {x = 0} 1. 1 4
P {x = 0 / y = 0} = = 2 = = 0.8
P { y = 0} 1. 1 + 0.25 ×
1 5
2 2

51. An M-level PSK modulation scheme is used to transmit independent binary digits over a
band-pass channel with bandwidth 100 kHz. The bit rate is 200 kbps and the system
characteristic is a raised-cosine spectrum with 100% excess bandwidth. The minimum value
of M is ________.
Answer: 16
1
Exp: Bandwidth requirement for m-level PSK = (1 + α )
T
[Where T is symbol duration. α is roll of factor]
1
⇒ (1 + α ) = 100 × 103
T
α = 1 [100% excess bandwidth ]
1
⇒ ( 2 ) = 100 × 103
T
Bit duration
2
⇒T= 1
100 × 103 = = 0.5 × 10−5 = 5 × 10−6 sec
= 20 µ sec 200 × 10 3

Symbol duration 20 × 10−6 sec


Bit duration = ⇒ log 2 m = = 4 ⇒ M = 16
log 2 m 5 × 10−6

 India’s No.1 institute for GATE Training  1 Lakh+ Students trained till date  65+ Centers across India
27
EC-GATE-2014 PAPER-04| www.gateforum.com

52. Consider a discrete-time channel Y =- X + Z, where the additive noise z is signal-dependent.


In particular, given the transmitted symbol X ∈{ − a, +a} . at any instant, the noise sample Z is
chosen independently from a Gaussian distribution with mean β X and unit variance. Assume
a threshold detector with zero threshold at the receiver.
When β = 0, the BER was found to be Q(a) = 1 × 10-8
 1 ∞ 
 Q ( v ) = ∫ e − u / 2du, and for v > 1, use Q ( v ) ≈ e − v
2 2
/2

2π v

When β = −0.3, the BER is closest to


(A) 10-7 (B) 10-6 (C) 10-4 (D) 10-2
Answer: C
Exp: X∈[-a,a] and P(x = -a) = P(x = a) = ½

γ =X+Z → Received signal

Z ∼ N ( βX,1)
Q ( a ) = 1 × 10−8
1
− ( Z −βX )
2
1
fZ ( z ) = Q ( a ) ≈ e−ϑ
2
2 2
e

 −a + z if x = −a
γ=
 a + z if x = + a
H1 : x = + a
H 0 : x = −a
and Threshold = 0
1 −
1
( y − a (1+β ) )2
f γ ( y H1 ) = e 2


1 −
1
( y + a (1+β ) )2
fγ ( y H0 ) = e 2


BER :
Pe = P ( H1 ) P ( e H1 ) + P ( H 0 ) P ( e H 0 )

1 − 12 ( y − a (1+β) )2 1 − 12 ( y + a (1+β ))2
0
dy = Q ( a (1 + β ) )
1 1
= ∫ 2π
2 −∞
e dy +
2 −∫0 2π
e

β=0
Pe = Q ( a ) = 1 × 10−8 = e− a ⇒ a = 6.07
2
2

β = −0.3
Pe = Q ( 6.07 (1 − 0.3) ) = Q ( 4.249 )
− ( 4.249 ) 2
2
Pe = e = 1.2 × 10−4
Pe  10−4.

 India’s No.1 institute for GATE Training  1 Lakh+ Students trained till date  65+ Centers across India
28
EC-GATE-2014 PAPER-04| www.gateforum.com

53. The electric field (assumed to be one-dimensional) between two points A and B is shown. Let
ψ A and ψ B be the electrostatic potentials at A and B, respectively. The value of ψ B − ψ A in
Volts is ________.

40 kV / cm
20 kV / cm

0 kV / cm
A B
5µ m
Answer: -15
Exp: A B

( 0 kV / cm, 20kV / cm ) (5 × 10 −4
kV / cm, 40 kV / cm )

40 − 20
−4 (
E − 20 = x − 0 ) ⇒ E = 4 × 104 x + 20
5 × 10
( 4 × 10 x + 20 ) dx
B 5×10−4 / cm
VAB = − ∫ E.dl = − ∫ 4
A 0
5×10−4
 
= − ( 2 × 104 × 25 × 10−8 + 20 × 5 × 10−4 )
x2
= −  4 × 104 + 20x 
 2 0
= − ( 50 × 10−4 + 100 × 10−4 ) = −150 × 10−4 kV
⇒ VAB = −15V


54. Given F = zaˆ x + xaˆ y + yaˆ z . If S represents the portion of the sphere x 2 + y 2 + z 2 = 1 for
 
z ≥ 0 , then ∫ ∇ × F . ds is ___________.
S

Answer: 3.14
   
Exp: ∫ ∇ × F.ds = ∫
S C
F.dr(u sin g stoke 's theorem and C is closed curve i.e.,

x 2 + y 2 = 1, z = 0
⇒ x = cos θ, y = sin θ and θ : 0 to 2π

= ∫ zdx + xdy + ydz


C


= ∫ xdy = ∫ cos θ ( cos θ dθ )
C 0


1 sin 2θ 
= θ +  = π  3.14
2 2 0

 India’s No.1 institute for GATE Training  1 Lakh+ Students trained till date  65+ Centers across India
29
EC-GATE-2014 PAPER-04| www.gateforum.com

55. If E = − ( 2y3 − 3yz 2 ) xˆ − ( 6xy 2 − 3xz 2 ) yˆ + ( 6xyz ) zˆ is the electric field in a source free
region, a valid expression for the electrostatic potential is
( A) xy 3 − yz 2 ( B) 2xy 3 − xyz 2 (C) y3 + xyz 2 ( D) 2xy 3 − 3xyz 2
Answer: D
Exp: Given E = − ( 2y3 − 3yz 2 ) a x − ( 6xy 2 − 3xz 2 ) a y + 6xyz.a z
By verification option (D) satisfy
E = −∇V

 India’s No.1 institute for GATE Training  1 Lakh+ Students trained till date  65+ Centers across India
30

You might also like